Sie sind auf Seite 1von 58

CPIM Exam Content Manual

This manual is in effect from January 1, 2008,


through December 31, 2008.

The references in this manual have been selected solely on the basis of their educational value to the APICS
CPIM certification program and on the content of the material. APICS does not endorse any services or other
materials that may be offered or recommended by the authors or publishers of books and publications listed in
this manual.
2008 APICS The Association for Operations Management
5301 Shawnee Road
Alexandria, VA 22312-2317
(703) 354-8851
(800) 444-2742
Fax: (703) 354-8106
No portion of this document may be reproduced under any circumstances. CPIM is a registered trademark of
APICS.
Stock #09051

CONTENTS

Page

Introduction
Letter to Candidates .......................................................................................................... i
Introduction ...................................................................................................................... iii
About the CPIM Examinations ......................................................................................... iii
Code of Ethics ................................................................................................................. vi
CPIM Certification Practice Questions ............................................................................ vii
Additional Resources for CPIM Candidates .................................................................... vii
CFPIM ........................................................................................................................... viii
Scope of the Subject Matter
Basics of Supply Chain Management .............................................................................. 1
Master Planning of Resources ........................................................................................ 10
Detailed Scheduling and Planning ................................................................................. 16
Execution and Control of Operations ............................................................................. 27
Strategic Management of Resources ............................................................................. 34
Answers to Sample Questions
Basics of Supply Chain Management Answers ............................................................. 41
Master Planning of Resources Answers ........................................................................ 42
Detailed Scheduling and Planning Answers .................................................................. 42
Execution and Control of Operations Answers .............................................................. 44
Strategic Management of Resources Answers .............................................................. 45

Contents

CPIM Exam Content Manual

Dear Candidate:
For more than 30 years, the Certified in Production and Inventory Management (CPIM) program has been
recognized as the international standard for individual assessment in the field of production and inventory
management. Initiated by APICS in 1973, it provides a common basis for individuals and organizations to
evaluate their knowledge of this evolving field. APICS has administered more than 820,000 tests in 40 countries,
and nearly 87,000 people have earned the CPIM designation3,000 of these at the Fellow level.
The mission of the CPIM program is to be the premier professional certification for operations management that
tests the candidates knowledge and understanding of the principles and practice of production and inventory
management. The CPIM program is designed to educate individuals in the concepts, tools, terminology, and
integration of topics within the operations management function and test candidates in-depth knowledge of
these concepts. APICS has worked to ensure that CPIM exams are consistently reliable and that the highest
professional standards are used to develop and administer the program.
Because organizations operate in a dynamically changing international world, the APICS body of knowledge
continues to grow to include recognized concepts and tools to improve organizational competitiveness. The
CPIM Exam Content Manual is updated regularly to reflect these changes in the body of knowledge and to
assist candidates in their understanding of the scope of material covered in the program.
Using a typical business process orientation, the CPIM program integrates individual modules in a progression
of increased understanding as follows:
1. Entry moduleBasics of Supply Chain Management: Terminology and basic concepts related to
managing the complete flow of materials from suppliers to customers.
2. Core competency modulesMaster Planning of Resources, Detailed Scheduling and Planning,
Execution and Control of Operations: Available methodologies and techniques to drive processes and
the application of these techniques.
3. Capstone moduleStrategic Management of Resources: Choosing the appropriate structures and
methodologies to achieve organizational strategic objectives and an understanding of the integration of
operations within the context of the organization.
The integration is presented here and represents a suggested sequence for the exams. Following is a summary
of each of the CPIM modules.

CPIM Program

Basics of Supply Chain


Management
An introduction for production and inventory
management personnel and CPIM candidates, this
module provides basic definitions and concepts for
planning and controlling the flow of products or
services into, through, and out of an organization. It
explains fundamental relationships among the
activities that occur in the supply chain from
suppliers to customers. In addition, the module
covers types of manufacturing and service systems,
forecasting, master planning, material requirements
planning, capacity management, production activity
control, purchasing, inventory management,
distribution, quality management, and Just-inTime/lean operations.

Introduction

Master Planning of Resources


In this module, candidates explore processes used to develop sales and operations plans; identify and assess
internal and external demand and forecasting requirements; and gain an understanding of the importance of
producing achievable master schedules that are consistent with business policies, objectives, and resource
constraints. The module focuses on developing and validating a plan of supply, relating management of demand
to environment, and developing and validating the master schedule.
In addition, the module encompasses concepts for transforming sales, marketing, and business requirements
into a feasible and economic operations plan in various business environments. It also addresses concepts and
methodologies for managing projected and actual demands from distribution networks and external customers.
Finally, the module presents methods for integrating sales and operations plans, demand forecasts, and
customer demand into a specific master schedule.

Detailed Scheduling and Planning


In this module, candidates focus on the various techniques for material and capacity scheduling. The module
includes detailed descriptions of material requirements planning (MRP), capacity requirements planning (CRP),
inventory management practices, and procurement and supplier planning. New techniques are discussed,
including material and capacity constrained scheduling, which are applicable to multiple industries including the
service sector. Candidates will also become familiar with supplier partnerships, lean inventory principles, and
outsourcing strategies and techniques.

Execution and Control of Operations


This module focuses on three main areas: execution of operational plans and schedules, control of the release
of work to the operational area assigned, and reporting the measurement of the performance. The module
explains techniques for scheduling and controlling operations. ECO deals with the execution of quality initiatives
and continuous improvement plans. Finally, the module presents techniques for collecting data and measuring
performance for feedback.

Strategic Management of Resources


In this module, candidates explore the relationship of existing and emerging processes and technologies to
operations strategy and supply chain-related functions for both manufacturing and service organizations. The
module addresses three main topics: aligning resources with the strategic plan, configuring and integrating
operating processes to support the strategic plan, and implementing change. For maximum comprehension of
course content, candidates should be familiar with the information and concepts presented in the other CPIM
modules before taking this course.
The CPIM program continues to evolve, incorporating relevant and current concepts and techniques into the
body of knowledge, such as supply chain management, lean, service industries, globalization, theory of
constraints, project management, outsourcing, critical chain, and new approaches in quality management.
CPIM is an outstanding educational program and APICS relies on your comments and suggestions to maintain
and improve the program for future candidates. We wish you success in your pursuit of your operations
management knowledge.

Paul Pittman, Ph.D., CFPIM


CPIM Program Chair
Indiana University Southeast

ii

CPIM Exam Content Manual

Introduction
This exam content manual provides assistance for
those studying in the production and inventory
management field, developing and conducting
education courses and workshops, and preparing
for the certification examinations. The objective of
this manual is to outline the CPIM body of
knowledge, which the APICS Body of Knowledge
Council has organized into five modules:
Basics of Supply Chain Management
Master Planning of Resources
Detailed Scheduling and Planning
Execution and Control of Operations
Strategic Management of Resources.
In this manual, each module begins with a
statement of the scope of the subject matter,
followed by a descriptive outline of the curriculum.
Key terminology for the particular module is
provided, as well as a bibliography of primary and
additional references. Each module concludes
with sample questions typical of those that appear
on the examinations. The correct answers for the
sample questions, with brief explanations of why
they are correct, appear at the end the manual.

About the CPIM


Examinations
All CPIM exams are administered using a lineartesting approach. Linear testing is a proven
testing methodology based on the same concept
as traditional paper-and-pencil exams. Candidates
answer a predetermined number of questions to
assess their knowledge in key areas.
Each of the CPIM exams (except Basics) consists
of 75 multiple-choice questions. The Basics exam
consists of 105 multiple-choice questions. There
is a three-hour time limit for each CPIM exam.
For more information about testing and
registration policies and procedures, call APICS
Customer Support at (800) 444-2742 or (703)
354-8851 and request a current CPIM
Certification Bulletin. Candidates who plan to test
in North America should request stock #09053,
and candidates testing outside North America
should order stock #09055.

The recommended procedure for mastering the


subject matter is to review the curriculum outline,
which defines the material, and then to study each
topic, using the references. Candidates should
understand the definitions of the key terms in the
curriculum outline, as well as the outlined
techniqueswhy and how to apply them and
which ones to select.
New developments in the state of the art may be
described in current literature. Sufficient
references are given for each topical area to
provide different approaches to material covered
in each module and different styles of presenting
it. Reading the available APICS periodical
material, including APICS magazine, Production
and Inventory Management Journal, and the
Journal of Operations Management, will help you
to maintain an awareness of changes in the state
of the art.

Introduction

iii

Question Format
Several types of questions are used in CPIM
examinations. (This manual includes examples of
many of these types of questions in the sample
questions. A review of all of these types of
questions is recommended to ensure maximum
preparedness.) For each question, one and only
one of the four choices represents the correct
answer. It is important that you make a choice
even if you are not certain of the correct answer,
because the omission of an answer will be
counted the same as an incorrect answer.
Examples 1 and 2 are samples of an incomplete
statement or a question, followed by four
suggested completions or answers. In Example 1,
you are to choose the one optionA, B, C, or D
that completes the statement correctly. In
Example 2, you are to choose the option that best
answers the question.
Example 1: The key to a successful
production plan is
(A) capacity requirements planning
(B) material requirements planning
(C) dynamic priority planning
(D) adequate production capacity
(The correct answer is D.)
Example 2: Which of the following
approaches allows MRP techniques to be
used for planning and controlling independent
demand items?
(A) pegged orders
(B) two-bin system
(C) time-phased order point
(D) reorder point
(The correct answer is C.)
Example 3 is different. All the choices given are
correct EXCEPT one. You are to choose the
option that does not correctly complete the
statement.
Example 3: Group technology identifies the
similarities among all of the following EXCEPT
(A) costs
(B) parts
(C) shapes
(D) processes
(The correct answer is A.)
Example 4, another type of multiple-choice
question, includes two or more statements or
possibilities. The question and the statements or
possibilities are always followed by four answer
choices labeled A, B, C, or D. When answering
multiple-choice questions of this type, read the
iv

CPIM Exam Content Manual

question and the statements or possibilities


carefully. Then determine whether each of
statements I through IV is true or false.
Next, look at the four choices. In Example 4, you
should pick option A if you believe statements I
and III are true. You should pick option B if you
believe statements I and IV are true. You should
pick option C if you believe statements II and III
are true. You should pick option D if you believe
statements II and IV are true.
Example 4: If a company changes from
make-to-stock to assemble-to-order, the
effects on inventory levels are which two of
the following?
I. Lower finished-goods inventory
II. Higher finished-goods inventory
III. Lower work in process
IV. Higher work in process
(A) I and III
(B) I and IV
(C) II and III
(D) II and IV
(The correct answer is B.)
Example 5 is similar to Example 4, but the number
of possible combinations is greater. The best
strategy for answering these questions is to
consider each statement, decide whether it is true,
and then search for the correct combination. If the
combination you seek is not given, reconsider
each statement carefully.
Example 5: Management policies and
decisions about which of the following have a
direct impact on investment?
I. Customer service levels
II. Intracompany transportation modes
III. Placement of distribution centers
IV. Types of production processes
(A) I and II only
(B) III and IV only
(C) I, II, and IV only
(D) I, II, III, and IV
(The correct answer is D.)
Examples 6 and 7 ask for a judgment or
evaluation of the MOST or LEAST appropriate
choice. The judgment is not one persons opinion,
but is the accepted choice according to the APICS
body of knowledge. Example 6 asks for the MOST
appropriate choice. Example 7 calls for the
LEAST appropriate choice.

Example 6: The MOST significant


disadvantage of aggregating demand data
before they are stored is that
(A) information about demand is lost
(B) there is risk of input error in the
aggregation process
(C) data will usually be inconsistent with
financial information
(D) the processing time required to aggregate
is extensive
(The correct answer is A.)
Example 7: Which of the following lot-sizing
calculations would be LEAST sensitive to
changes in unit costs?
(A) least total cost
(B) period order quantity
(C) part period balancing
(D) lot-for-lot
(The correct answer is D.)

Taking the Test


You should read each question carefully and
thoroughly. Take care to avoid assuming
information not given, assuming you know what is
being asked without reading the question
completely, or second guessing the question.
Every effort has been made to avoid misleading
wording and to provide sufficient information for
each question.
Each of the questions is based on the outline in
this exam content manual. The test is designed to
evaluate a candidates knowledge of the subject
matter. Therefore, the key to success is a
thorough understanding of the subject matter.
When you start your exam, read all the directions
carefully. Be sure you understand the directions
before you begin to answer any questions.
Read each question carefully. If a question
includes stimulus material such as a table or
graph, be sure to study it before you answer the
question. Choose the best answer from the
choices given. You may believe that a different
answer is correct but was not provided. Care has
been taken to avoid misleading choices. Do not
look for hidden tricks or exceptions to the norm.
All questions are based on the current APICS
body of knowledge as defined in the exam content
manual.
Once you begin the test, approach the questions
in order, but do not waste time on those that are
unfamiliar or seem difficult to you. Go on to the
other questions and return to the difficult ones
later if you have time. If you have some

knowledge about a particular question, you may


be able to eliminate one or more choices as
incorrect. Your score on the test will be based on
the number of questions you answer correctly,
with no penalty for incorrect answers; therefore, it
is to your advantage to guess rather than not
answer a question. Avoid changing an answer
unless you are absolutely certain that you marked
the wrong answer the first time.

Interpreting Test Scores


The CPIM scaled score range is 265330:
265299 Fail
300330 Pass
For each examination, you receive a score for the
total test. All candidates will also receive
diagnostic information on their performance. A
total scaled score of 300 or higher is required to
pass an exam.

Suggested Study Materials


Bibliography Section. Each of the modules in this
manual contains a bibliography section that lists
primary and additional references. Although all of
these references contain excellent material that
will assist in test preparation, the primary
references usually contain the most
comprehensive treatment of the subject matter
included in the module. Those concepts and
subjects covered by the additional references,
limited to the scope of the curriculum outline, are
a part of the APICS body of knowledge and are
subject to examination questions.
A candidate may discover that the material
covered in the chapters of one primary reference
duplicates material covered in another primary
reference. Both sources are included as primary
references to allow candidates some discretion in
selecting test preparation materials that they find
accessible and understandable. For instance, a
candidate who uses a specific reference in
preparing for a certification exam that he or she
passed may feel comfortable using that same
reference to prepare for other certification exams.
In deciding if a single primary reference is
sufficient, candidates should assess their own
level of knowledge against both the descriptive
examination specifications and the detailed topic
list contained in the curriculum outline. If there are
any areas of weakness, the candidate should
consult another primary reference or an additional
reference as part of the test preparation process.

Introduction

It is not practical to list all texts that contain


excellent material. The serious student of
operations management who wishes to stay
current with the state of the art will take advantage
of publications such as those listed in the APICS
Educational Resources Catalog (stock #01041), a
comprehensive catalog featuring APICS reference
materials. To receive the catalog, call APICS
Customer Support at (703) 354-8851 or (800)
444-2742.
Curriculum outline. The curriculum outline for
each module provides a detailed overview of the
major topics included in that module. Each major
topic is denoted by a roman numeral and is
followed by a list of the references that are
particularly relevant to that topic.
th

APICS Dictionary. The APICS Dictionary, 12


edition, is an essential publication that applies to
the exam content manual and exams. Within the
profession, terminology varies among industries,
companies, and the academic community. Each
examination uses standard terminology as defined
in the APICS Dictionary. Recognizing the terms
and understanding their definitions are essential.
Reprints. The committee responsible for the
development of the exam content manual and the
examination selects articles that are particularly
applicable to the curricula and the candidates
preparation for the examination. These articles
are then reprinted in module-specific collections.
These reprints are included in the primary
references for each module.
Other resources. In addition to the references
cited, it may be helpful for you to pursue chaptersponsored courses, college courses, APICS
seminars, or self-study courses as a means of
learning the body of knowledge that is tested in
the certification program. A wide variety of
courses is available. As with any other investment
of time and money, you should research various
courses before investing in one. Please visit the
Chapter Locator on the APICS Web site
(www.apics.org) or call APICS Customer Support
at (703) 354-8851 or (800) 444-2742 to obtain
contact information for your local chapter.
Please refer to the most current APICS
Educational Resources Catalog for further
information.

Terminology
In studying for the CPIM certification, candidates
may discover multiple terms used to denote the
same technique. Examples of this include sales
vi

CPIM Exam Content Manual

and operations planning versus production


planning and master production schedule versus
master schedule. APICS has attempted to provide
consistency across all modules with recognized
and preferred terminology. However, synonyms
are often used by authors in the various
references used to compile the body of
knowledge. Candidates are encouraged to be
familiar with all terms and concepts listed within
each outline and key terminology section, using
the APICS Dictionary as the primary guide for
definitions. Supplemental glossaries are offered in
this manual wherever the committees feel that
additional information is needed.

Code of Ethics
When you start an examination, you will be asked
to pledge to abide by the APICS Code of Ethics.
Once certified, you pledge to continue your
education to increase your contribution to the
operations management profession. After
achieving the higher level of certification (CFPIM),
you pledge also to share your CPIM knowledge
with others by participating in APICS research and
educational activities at local, regional, and
national levels.
The Code of Ethics:
To maintain and improve sound business
practices and foster high standards of
professional conduct.
To hold in professional confidence any
information gained of the business of a
fellow members company, and to refrain
from using such information in an
unethical manner.
To seek success without taking unfair
advantage or utilizing questionable acts
that would compromise ones self-respect.
To neither engage in nor sanction any
exploitation of ones membership,
company, or profession.
To encourage and cooperate in the
interchange of knowledge and techniques
for the mutual benefit of the profession.
To be careful with ones criticisms and
liberal with ones praise; to build and not
to destroy.
Whenever a doubt arises as to the right or
ethics of ones position or action, to
resolve such doubt according to generally
accepted standards of truth, fair dealing,
and good taste.
To maintain high personal standards of
moral responsibility, character, and
business integrity.

To uphold the high ideals of the


association as outlined in the bylaws.

CPIM Certification
Practice Questions
The practice questions for each module include a
sampling of questions covering the APICS body of
knowledge and display the various test question
formats used in the exams. The questions were
selected or reviewed by the individual committees
of the Body of Knowledge Council.
Basics of Supply Chain Management
Practice Questions Booklet
Stock #09205
Practice Questions CD-ROM
Stock #09215
Master Planning of Resources
Practice Questions Booklet
Stock #09209
Practice Questions CD-ROM
Stock #09216
Detailed Scheduling and Planning
Practice Questions Booklet
Stock #09211
Practice Questions CD-ROM
Stock #09217
Execution and Control of Operations
Practice Questions Booklet
Stock #09212
Practice Questions CD-ROM
Stock #09218
Strategic Management of Resources
Practice Questions Booklet
Stock #09213
Practice Questions CD-ROM
Stock #09219
Complete Set of Practice Questions Booklet
Stock #09210

CPIM References
The APICS Certification Committees have
identified a number of references for each CPIM
module. These references include texts and
reprints of pertinent articles and case studies. For
a complete list of CPIM references, visit our Web
site at www.apics.org, or call APICS Customer
Support at (800) 444-2742 or (703) 354-8851.

CPIM Review Courses


Available for each module of the CPIM process,
the review courses are designed for classroom
review of the key principles and concepts for each
content area. The review course participant
workbook is not a stand-alone reference nor
comprehensive single source and should be used
only by a participant attending an instructor-led
review course. For more information, visit our
Web site at www.apics.org or call APICS
Customer Support at (800) 444-2742 or (703)
354-8851 and request a complimentary APICS
Educational Resources Catalog (stock #01041).

Independent Study Courses


APICS correspondence courses offer
professionals a unique home-study alternative.
Correspondence courses are designed and
conducted for APICS by the MGI Management
Institute. For more information on course
availability, call APICS Customer Support at (800)
444-2742 or (703) 354-8851.

Educational Programs
Complete Set of Practice Questions CD-ROM
Stock #09220
Note: A Practice Questions CD-ROM is included
in each participant workbook for each module.
Although these practice questions may be helpful
in preparing for exams, they should be used only
to augment a comprehensive program of study
and preparation.

Additional
Resources for CPIM
Candidates
APICS offers a number of resources to help
individuals prepare for the CPIM examinations.

APICS offers a variety of educational programs,


including workshops in operations management
essentials and integrated resource management
and an international conference and exposition.
For a complete list of APICS learning
opportunities and information on course
availability, call APICS Customer Support at (800)
444-2742 or (703) 354-8851.

Chapter Certification Review


Classes
Many chapters offer certification review classes.
For more information, contact your local chapter.
If you need a list of chapters in your area, visit our
Web site at www.apics.org, or call APICS
Customer Support at (800) 444-2742 or (703)
354-8851.

Introduction

vii

For a complete list of APICS resources, call


APICS Customer Support and request a free copy
of the APICS Educational Resources Catalog
(stock #01041).

To obtain copies and a more detailed explanation


of the CFPIM guidelines and application, contact
APICS Customer Support at (800) 444-2742 or
(703) 354-8851 and request stock #09052.

CFPIM

Certification Maintenance
Continuing Professional
Development

The distinguishing characteristic of a Certified


Fellow in Production and Inventory Management
(CFPIM) is the willingness to share acquired
knowledge with others through presenting,
teaching, publishing, and participating in APICS
educational activities. This knowledge sharing
must take place above and beyond a candidate's
normal job duties and be directly related to the
CPIM body of knowledge.
To obtain the CFPIM designation, an application
form must be filled out and submitted to APICS
Headquarters. Points are awarded based on the
following criteria: CPIM exams passed,
presentations, high scores on CPIM exams,
published works, classroom teaching, and various
volunteer/practitioner activities.

viii

CPIM Exam Content Manual

To promote professional growth and lifelong


learning, CPIM and CFPIM designees must
complete the Certification Maintenance program
every five years. Complete details on how to
maintain the CPIM and CFPIM designations will
be mailed to candidates upon successful
completion of the certification requirements. To
learn more about the Certification Maintenance
program, call APICS Customer Support at (800)
444-2742 or (703) 354-8851 and request a copy
of the Certification Maintenance Bulletin (stock
#09033) or visit the APICS Web site at
www.apics.org and download a copy of the
bulletin.

Basics of Supply
Chain Management
Effective for January 1, 2008December 31,
2008, Exams.
EXAMINATION COMMITTEE
Jim Caruso, CPIM, CSCP (Chair), Covidien
Carol Bulfer, CPIM, Parker Hannifin Corp.
Jim Cox, Ph.D., CFPIM, CIRM, Jonahs Jonah,
TOCICO-certified, Professor Emeritus,
University of Georgia
William Leedale, CFPIM, CIRM, IFS
Mike Roman, CPIM, Manufacturing Practices, Inc.
Angel A. Sosa, CFPIM, University of Puerto Rico
at Bayamon

Scope of the Subject Matter


This module covers basic concepts in managing
the complete flow of materials in a supply chain
from suppliers to customers. It emphasizes basic
terminology and covers relationships among
activities in the supply chain. Knowledge of the
material in this module is assumed as a
prerequisite for the other CPIM modules, which
cover similar topics but in much greater depth.
The Basics module is divided into four major
sections:
1. Business Wide Concepts
2. Demand Planning
3. Transformation of Demand into Supply
4. Supply
The first section of the content outline covers
basic business wide concepts, including an
understanding of the various production
environments used for the transformation process.
Three common companywide management
approachesEnterprise resources planning
(ERP), Just-in-Time (JIT/Lean), and total quality
management (Quality Systems)and their
interrelationships are presented. Some financial
fundamentals are also covered.
The second section of the outline covers demand
planning, including a basic understanding of
markets and customer expectations, the definition
of value, and a fundamental overview of demand
forecasting. This information about demand
serves as the primary input to the third section of
the outline, which includes the design,
management, and control of the transformation
process itself.

The first part of the third section includes the


design of products, processes, and information
systems used for planning. The rest of the section
includes fundamentals of planning systems,
execution controls, and performance measures.
The fourth and final section of the content outline
is devoted to supply issues. Specifically, this
section covers concepts in inventory, purchasing,
and physical distribution.

Basics of Supply Chain


Management Curriculum
Four topics have been used to organize the
domain of Basics of Supply Chain Management.
The relative importance of a topic is not
necessarily reflected by its level in the outline. The
relative importance of these topics will vary
among industries, but for study purposes the
percentage figures given below can be used as
guidelines.
DIAGNOSTIC
PART

MAIN TOPIC

PERCENTAGE
OF EXAM

Business Wide Concepts

24%

II

Demand Planning

17%

III

Transformation of Demand
into Supply

32%

IV

Supply

27%

Curriculum Outline
I. Business Wide Concepts
This section of the outline covers basic
business wide concepts, including the various
production environments used for the
transformation process and financial
fundamentals.
Also in this section, three of the more
common companywide management
approaches (ERP, JIT/Lean, and Quality
Systems) are individually presented, together
with their interrelationships.
A. Organization Fundamentals: The basic
concept of a supply chain includes all
activities and processes used to provide a
product or service to the ultimate
customer. It also includes internal
organization dynamics, such as the
traditional conflicts between functional
areas.
Basics of Supply Chain Management

1. Elements of the supply chain


a. supplier
b. producer
c. distributor
d. retailer
e. customer
f. service and support
2. Internal organizational dynamics
B. Operating Environments: Environments
are the individual market and
manufacturing factors that combine to
determine the specific operating
conditions.
1. Definition and impact of
a. customer expectations
b. cumulative lead time
c. inventory
d. product life cycle
e. product design
2. Process choices
a. project
b. lot/batch/intermittent
c. line/repetitive
d. continuous
3. Production environment
a. design/engineer-to-order
b. make-to-order
c. assemble- or package-to-order
d. make-to-stock
e. remanufacture
C. Financial Fundamentals: Basic financial
statements define the financial reporting
common to most businesses. Underlying
costs and analysis terms provide further
understanding of statement information
and often serve as the basis for
management decisions.
1. Statements
a. balance sheet
b. income statement
c. cash flow statement
2. Costs
a. cost of goods sold
b. general and administrative
c. fixed and variable costs
3. Analysis
a. cash flow
b. profit
c. margin
d. inventory turns
D. Enterprise Resources Planning (ERP):
ERP is a fully integrated planning and
control information system that serves as
a master game plan for the business.
(MRP II is a subset of ERP for
manufacturers).
1. Objectives
2. Key principles and characteristics
a. top-management planning tool

CPIM Exam Content Manual

b. integrated planning structure


c. cross-functional integration
d. closed loop (feedback)
e. what-if simulations
f. financial data
E. Just-in-Time (JIT/Lean): JIT and Lean can
be understood as philosophies
embodying certain objectives and as sets
of manufacturing techniques described by
specific characteristics.
1. Objectives
2. Key principles and characteristics
a. flow manufacturing
b. process flexibility
c. quality at the source
d. continuous improvement
e. supplier partnerships
f. employee involvement
g. total productive maintenance
h. pull systems
i. work cells
j. elimination of waste
3. Concepts of waste and value-added
activity
F. Quality Systems: Quality systems focus
on customer needs including a specific
set of tools to improve products, services,
and production processes through the use
of such techniques as employee
empowerment.
1. Objectives
2. Key principles and characteristics
a. problem-solving tools
b. costs of quality
c. customer focus
d. employee empowerment
e. process improvement
G. Impact of Environment on System Design
and Deployment: The production
environment can be significantly improved
by JIT/Lean and Quality Systems. This in
turn simplifies ERP system operation and
improves business performance.
1. JIT/Lean using ERP
2. Quality Systems impact
Reference: 1; 3 (chapters 1, 2, 14-16); 4

II. Demand Planning


This section covers demand planning,
including a basic understanding of markets
and customer expectations, the definition of
value, and a fundamental overview of demand
forecasting.
A. Marketplace-Driven: Customer needs and
alternative competitive sources determine
the type of product demand experienced
by supplying firms.

1. Customer
2. Competitors
3. Economy and regulatory policy
B. Customer Expectations and Definition of
Value: A firm must determine the primary
characteristics of products and services
that are attractive to customers. This
understanding can then be exploited
through appropriate market plans and
production processes.
1. Order winners/qualifiers
a. quality
b. delivery
c. pre-sale and post-sale service
d. price
e. flexibility
2. Marketing strategy
C. Customer Relationships Management: To
develop effective customer relationships,
ongoing interaction and communication
with customers is essential with both
product design and production deliveries.
1. Expanding product/service offerings
2. Design assistance
3. Information/communications
D. Demand Management: Demand
management is the recognition of
demand, either as forecast or actual
customer orders. Forecasting is complex,
but it involves a variety of standard terms.
Once recognized, demand is met through
a combination of inventories and the
master production schedule. Distribution
requirements planning (DRP) improves
demand management by directly linking
distribution system demand to central
demand.
1. Sources of demand
2. Forecast management
a. characteristics of demand
b. principles of forecasting
c. methods
d. forecast error measurement and
response
Reference: 3 (chapters 8, 9)

III. Transformation of Demand into


Supply
This section includes the design,
management, and control of the
transformation process itself.
The first part of this section includes the
design of products, processes, and
information systems used for planning.

The rest of the section includes fundamentals


of planning systems, execution controls, and
performance measures.
A. Design: Design affects product and
process, the resulting framework of
planning system parameters, and the
requirement for data appropriate in
source, content, and accuracy.
1. Manufacturing feasibility
a. new product design and
introduction
b. process choices
c. process flexibility
2. Planning parameters
a. manufacturing and purchasing
lead time
b. order quantity/lot size/batch
c. safety stock and/or capacity
3. Data sources and data accuracy
a. bills of material/formulas/recipes
b. planning bill of material
c. item master (material master)
d. routing/process
e. work center/flow line
4. Functional responsibilities
B. Capacity Management: The fundamentals
of capacity management include the
required resources, the various methods
of measuring capacity, the levels and
stages of planning capacity, and the
impact of bottlenecks and constraints.
1. Resources
2. Measuring capacity
3. Capacity planning
4. Bottlenecks/constraints
C. Planning (purposes, inputs, and outputs):
The purpose and structure of the planning
process are the basic inputs, data
sources, and necessary outputs.
Functional trade-offs are resolved in
effective higher-level planning. Other
plans at all levels must remain consistent
with one another and with the overall
plan.
1. Strategic planning and business
planning
2. Sales and operations (S &OP)
planning/production planning
3. Master scheduling and rough-cut
capacity planning (RCCP)
4. Material requirements planning (MRP)
and capacity requirements planning
(CRP)
5. Final assembly scheduling (FAS)
D. Execution and Control: Fundamental
definitions and structures of execution
and control techniques are widely known.
Understanding them requires a thorough
comprehension of scheduling and work-

Basics of Supply Chain Management

in-process techniques and how they


interrelate to meet the master production
schedule. Customer service activities and
quality assurance further affect execution
and control.
1. Customer service
a. order processing
b. order promising
c. push/pull
2. Linkages to the master schedule
3. Scheduling techniques
a. forward/backward
b. finite/infinite
4. Work-in-process
a. capacity control
b. priority control
c. production reporting
5. Quality assurance
a. measuring quality
b. process variation
c. process control
E. Performance Measurements: There are
several types of key performance
indicators , each with specific reasons for
their use. Some are strategic in nature,
while others are operational or tactical in
nature. It is important to have an
appropriate mixture of both types for an
effective evaluation of organizational
performance.
1. Strategic
2. Tactical
Reference: 1; 3 (chapters 2-6)

IV. Supply
This section is devoted to supply issues.
These include the concepts of inventory,
purchasing, and physical distribution.
A. Inventory: Inventory can be described by
its various categories and functions and
whether it meets dependent or
independent demand. There are several
approaches to inventory management,
order system interrelationships, and
inventory cost analysis.
1. Categories
a. raw materials
b. work-in-process
c. finished goods
d. service parts
e. maintenance, repair, and
operating supplies (MRO)
2. Functions
a. transit/pipeline
b. cycle/lot size
c. anticipation
d. safety stock
4

CPIM Exam Content Manual

3. Dependent versus independent


demand systems
4. Management
a. ABC analysis
b. tracking inventory
c. cycle counting
d. record accuracy
e. physical inventory
5. Order systems
a. time-based
b. quantity-based
6. Cost analysis
a. costs
b. economic order quantity (EOQ)
c. safety stock level versus
customer service
B. Purchasing: There are several important
components of purchasing, especially
regarding suppliers and order
management, and ultimately including
receiving and invoice payment.
1. Receiving and analyzing notification
of need
a. requisition
b. MRP output
2. Selecting suppliers
a. source (single/multiple)
b. supplier partnering
c. vendor-managed inventory
3. Supplier agreements
a. price
b. terms
c. delivery
d. quality
e. quantity
4. Order management
a. ordering/releasing
b. monitoring and controlling open
orders
c. receiving and accepting orders
d. approving invoices for payment
5. Monitoring supplier performance
C. Physical Distribution System: The
components of a physical distribution
system should be recognized, as well as
the interfaces with production, marketing,
and finance.
1. Transportation
2. Warehousing
3. Distribution inventory
4. Interfaces with production, marketing,
and finance
Reference: 1; 3 (chapters 7, 10-13); 4

Key Terminology
An understanding of the following list of terms is
recommended. The list is intended to be thorough,

but not exhaustive. The candidate is also


expected to be familiar with the definitions of
terms identified in the curriculum outline. A review
of the list and study of the topics in the outline
may reveal other important terms. Definitions of
these terms can be found in the APICS Dictionary,
12th edition, or in the references listed in the
bibliography.

Customer service
Customer service ratio
Cycle
Cycle counting
Cycle stock
Cycle time

D
Decoupling
Demand
Demand management
Demand pull
Demonstrated (measured) capacity
Dependent demand
Direct labor
Direct material
Discrete manufacturing
Dispatching
Distribution inventory
Distribution requirements planning (DRP)
Distribution resource planning (DRP II)
Distributor
Due date

A
ABC inventory control
Actual costs
Aggregate lead time
Anticipation inventories
Assemble-to-order
Assembly line
Asset
Available inventory
Available-to-promise (ATP)
Average cost per unit

B
Backflush
Backlog
Backorder
Backward scheduling
Balance sheet
Barcode
Batch
Batch processing
Bias (to be consistent with MAD and random
variation)
Bill of material (BOM)
Blanket orders
Bottleneck
Buffer
Bullwhip effect
Business plan

C
Capacity
Capacity available
Capacity management
Capacity required
Capacity requirements planning (CRP)
Carrying cost
Cash flow
Cellular manufacturing (work cells)
Changeover
Chase (demand matching) method
Closed-loop MRP
Competitive analysis
Component
Consignment
Constraint
Continuous flow (production)
Control chart
Cost of goods sold
Cost of (poor) quality
Cumulative lead time
Customer Relationship Management (CRM)

E
Economic order quantity (EOQ)
Efficiency
Electronic data interchange (EDI)
Employee empowerment
Employee involvement (EI)
Engineer-to-order
Enterprise Resources Planning
Expedite
Explode
Extrinsic forecast

F
Feedback
Fill rate
Final assembly schedule (FAS)
Finished goods
Finish-to-order
Finite forward scheduling
Finite loading
Firm planned order (FPO)
Five Ss
Fixed order quantity
Flexible manufacturing system (FMS)
Flow manufacturing
Flow shop
Fluctuation inventory
Forecast error
Forecasting
Formula
Forward scheduling

G
General and administrative expenses
Gross margin
Gross requirements

Basics of Supply Chain Management

Master production schedule (MPS)


Master schedule
Material requirements planning (MRP)
Materials management
Mean absolute deviation (MAD)
Move time
Multilevel bill of material
Multisourcing

I
Income statement
Independent demand
Infinite loading
Infinite scheduling
Input/output control
Inspection
Intermittent production
Interplant demand
In-transit inventory
Intrinsic forecast
Inventory
Inventory adjustments
Inventory control
Inventory issues
Inventory management
Inventory returns
Inventory turnover
Inventory turns
Item master record

N
Net requirements

O
On-hand balance
Open order
Operation
Order entry
Ordering costs
Order point
Order promising
Order qualifiers
Order quantity
Order winners
Overhead
Owners equity

J
Job costing
Job order
Job shop
Job shop layout
Job shop scheduling
Just-in-Time (JIT)

K
Kaizen
Kanban

L
Leading indicator
Lead time
Lead-time offset
Lean
Level of service
Level production plan (level schedule)
Liability
Line manufacturing (assembly line)
Load
Load leveling
Lot
Lot control
Lot-for-lot
Lot size
Lot-size inventory

M
Maintenance, repair, and operating supplies
(MRO)
Make-to-order
Make-to-stock
Manufacturing lead time
Manufacturing process
Manufacturing resource planning (MRP II)
Market driven
Marketing research
Market share
Master planning

CPIM Exam Content Manual

P
Package-to-order
Packing and marking
Parent item
Pegging
Performance measurement system
Perpetual inventory record
Physical distribution
Physical inventory
Pipeline inventory
Planned order
Planned order receipt
Planned order release
Planning horizon
Poka-yoke
Priority control
Process
Process layout
Product differentiation
Product family
Production activity control (PAC)
Production line
Production plan
Production planning
Productivity
Product layout
Product life cycle
Profit
Profit margin
Projected available inventory (balance)
Pull (system)
Purchase order
Purchase requisition
Purchasing
Push (system)

Quality
Quality at the source
Quality control
Queue

Takt time
Throughput time
Time bucket
Time fence
Theory of Constraints (TOC)
Total acquisition costs
Total productive maintenance (TPM)
Total quality management (TQM)
Tracking signal
Transit time
Transportation inventory
Trend
Two-bin system

R
Radio Frequency Identification (RFID)
Random-location storage
Random variation
Rated capacity
Raw material
Receipt
Record accuracy
Remanufacturing
Reorder point
Reorder quantity
Repetitive manufacturing
Replenishment lead time
Request for quotations
Required capacity
Requirements explosion
Requisition
Resource planning
Reverse Logistics
Rough-cut capacity planning (RCCP)
Routing
Run time

S
Safety stock
Sales and operations planning (S & OP)
Sales plan
Scheduled receipt
Scheduling
Seasonality
Service industry
Service parts
Service parts demand
Setup
Setup lead time
Shop calendar
Shop order
Simulation
Single-level bill of material
Single-source supplier
Six Sigma
Split lot
Standard costs
Standard hours (time)
Start date
Statistical process control (SPC)
Stockkeeping unit (SKU)
Stockout costs
Strategic plan
Subcontracting
Supplier
Supplier lead time
Supplier partnering (partnership)
Supplier Relationship Management (SRM)
Supply chain

U
Unit of measure
Utilization

V
Value added
Value analysis
Value stream mapping
Variance
Vendor
Vendor-managed inventory (VMI)
Visual review system

W
Wait time
Warehousing
Waste
Where-used list
Work cell
Work center
Work-in-process (WIP)
Work order

Supplemental Glossary
The following concepts not found in the APICS
Dictionary, will be helpful in preparing for the
Basics of Supply Chain certification examination.
Value stream mapping: Maps created to
document current and future processes. The
current state map identifying steps that add or do
not add value. A future state map then details how
to minimize or eliminate non-value added
processes.
Total acquisition cost: represents the total cost of
a purchased item that includes price per unit,
charges, handling, transportation and delivery
cost to get the product to production.

Bibliography
All test candidates should familiarize themselves
with the following primary references for this
examination. The text Introduction to Materials
Basics of Supply Chain Management

Management covers the majority of the material


for this module. The additional references provide
further coverage of some of the topic areas and
can enhance candidates understanding of the
body of knowledge. Those concepts and subjects
covered by the additional references, limited to
the scope of the curriculum outline, are a part of
the APICS body of knowledge and are subject to
examination questions. All of the references are
available from the APICS Bookstore.

(C) a product that has the highest profit


margin
(D) a product that generates the highest
dollar sales volume
3. Which of the following is the primary purpose
of forecasting?
(A) projecting product demand
(B) estimating safety stock
(C) determining resource requirements
(D) loading the master production schedule

Primary References
1. APICS CPIM Basics of Supply Chain
Management Reprints, 2006.
2. APICS Dictionary, 12th ed., 2008.
3. Arnold, J.R. Tony, S.N. Chapman and LM.
Clive, Introduction to Materials Management,
6th ed., Prentice Hall, 2008.

Additional Reference
4. Feld, W.M., Lean Manufacturing: Tools,
Techniques, and How to Use Them, St. Lucie
Press, 2001.

Sample Questions
Following are 10 questions that are similar in
format and content to the questions on the exam.
These questions are intended for practicethat
is, to enable you to become familiar with the way
the questions are asked. The degree of success
that you have in answering these questions is not
related to your potential for success on the actual
exam and should not be interpreted as such.
Read each question, select an answer, and check
your response with the explanation on page 41.
1. Which of the following situations reflects the
best supplier/customer partnership?
(A) the supplier commits to the customers
requests.
(B) the company buys a part from the supplier
even though the company could make the
part.
(C) the customer and supplier implement
electronic data interchange.
(D) the customer and supplier share problemsolving responsibilities.
2. Which of the following is the most appropriate
definition of an order winner?
(A) a product characteristic that causes
customers to choose the product over that
of a competitor
(B) a product characteristic that qualifies the
product to compete in a market
8

CPIM Exam Content Manual

4. The production plan is used to establish which


of the following?
(A) customer service level
(B) gross sales volume
(C) the business plan
(D) product family rates of production
5. Order promising uses which of the following
elements of the master schedule?
(A) projected available balance
(B) available-to-promise
(C) safety stock
(D) forecast demand
6. Monitoring the statistical process control of an
established, stable process should be done by
which of the following?
(A) process operators
(B) supervisors
(C) manufacturing engineers
(D) quality engineers
7. Which of the following types of inventory is
used to protect against variations in supply
and/or demand?
(A) cycle stock
(B) common stock
(C) safety stock
(D) seasonal stock
8. Dependent demand items are most accurately
described as which of the following?
(A) finished goods awaiting final sale
(B) service parts
(C) products with a market-sensitive price
(D) raw materials, component parts, and
subassemblies
9. In backward scheduling, which of the following
is used as the first date to determine the
schedule?
(A) early start date
(B) late start date
(C) start date
(D) due date

10. The bill of material in a material requirements


planning (MRP) system serves which of the
following functions?
(A) provides the basis for the gross-to-net
calculation
(B) guides the explosion process
(C) guides the assembly process
(D) provides lot-sizing information
(Answers listed on page 41.)

Basics of Supply Chain Management

Master Planning of
Resources
Effective for January 1, 2008December 31,
2008, Exams.
EXAMINATION COMMITTEE
Mark K. Williams, CFPIM, CSCP, (Chair), Williams
Supply Chain Group, Inc.
Lynn Boyd, Ph.D., CPIM, University of Louisville
Murray R. Olsen, CFPIM, CIRM, ATK Launch
Systems
Josie Vigil, CPIM, CSCP, B. Braun Medical, Inc.

Scope of the Subject Matter


Please read the introductory material in this manual
for essential information about the examination.
This module evaluates knowledge of both supply
and demand planning for mid- to long-term
independent demand. Major topics include demand
management, sales and operations planning, and
master scheduling. It addresses both priority and
capacity planning.
Demand Management is the function of recognizing
all demands for goods and services to support the
marketplace. It includes forecasting, order servicing
and customer relationship management, as well as
distribution planning.
Sales and Operations Planning is a process that
brings together all the plans for the business (e.g.,
operations, sales, sourcing, marketing, product
development, and finance). The result is an
integrated set of plans for each product family.
Master Scheduling is the process of disaggregating
the production plan into an executable schedule
which links shipment of customer orders to material
management.
In each of these areas collaboration within the
organization and with direct customers and
suppliers will be addressed.

Master Planning of Resources


Curriculum
Three main subject topics have been used to
organize the domain of Master Planning of
Resources. A topics relative importance is not
necessarily reflected by its level in the outline.
Relative importance of these topics varies from
industry to industry, but for study purposes the
10

CPIM Exam Content Manual

percentage figures given below can be used as


guidelines.
DIAGNOSTIC
PART

MAIN TOPIC

PERCENTAGE
OF EXAM

Demand Management

30%

II

Sales and Operations


Planning

25%

III

Master Scheduling

45%

Curriculum Outline
I. Demand Management
This section addresses the interrelationship of
strategic and business planning with the
management of demand. The three primary
aspects of demand management (forecasting,
order servicing/customer relationship
management, and distribution planning) are
addressed. Exam questions covering this area
will include elements related to the following:
A. General Concepts and Purposes:
Successful planning requires an
understanding of how components,
concepts, and linkages of the strategic
plan, the business plan, and the master
planning of resources interact. Knowledge
in this area encompasses:
1. The planning hierarchy
2. Relating the planning process to the
environment
B. Forecasting Demand: Effective demand
forecasting requires an understanding of
the uses of a forecast, methods of
forecasting, fitting a forecast to the
situation, and tracking its performance over
time. Knowledge in this area encompasses:
1. Demand forecasting concepts
2. The relationship between the timeliness
and accuracy of the data and the
requirements of the forecast
3. Management considerations related to
forecast selection
4. How to select a time horizon and
interval, and a level of aggregation
5. The fit of basic qualitative techniques
6. The fit of basic intrinsic quantitative
techniques. Specific techniques that
should be understood include:
a. Simple averages
b. Single level exponential smoothing
c. Time Series Decomposition
7. The fit of basic extrinsic quantitative
techniques
8. Evaluating forecasting performance

C. Management of the Customer Interface:


Effective management of the customer
interface requires an understanding of how
to make realistic order delivery promises
and maintain positive customer
relationships. This area of the exam covers
topics related to ensuring effective
customer communication; handling actual
orders; and measuring the performance of
your order management system.
Knowledge in this area encompasses:
1. Customer Relationship Management
(CRM) concepts
2. How to set customer service policies
safety stock, and performance target
3. Methods for calculating and using
Available-to-promise (ATP) and
Capable-to-promise (CTP) quantities
4. How to maintain effective customer
communications
5. Techniques for measuring order
delivery performance
D. Distribution Planning: Effective distribution
planning requires being aware of the
various methods for disseminating products
or services to customers and the relative
strengths and weaknesses of those
alternatives. Knowledge in this area
encompasses:
1. The various distribution strategies and
network configurations
2. How to select between various
transportation methods
3. Where to locate inventory within the
network
4. How to link the distribution network to
customer demand and master
scheduling
5. How to measure the performance of the
distribution system
References: 1; 2; 3 (chapters 1, 3-9, 11, 12); 4
(chapters 2, 8); 5 (chapters 1, 2, 5, 8); 6
(chapter 4); 7; 8 (chapters 1, 16); 9 (chapters 5,
11)

II. Sales and Operations Planning


This section addresses the processes,
concepts, and techniques used to link strategic
goals to operations and coordinate the various
planning efforts of the functional areas (i.e.,
operations, sales, sourcing, product
development, marketing, and finance) in a
variety of business environments. Exam
questions covering this area will include
elements related to the following:
A. General Concepts and Purpose: Sales and
operations planning (S&OP) coordinates

the various business functions to ensure


they support the overall organization
strategy and enhance customer service. At
the center of the process are two
fundamental issues. First, what is the best
way to balance supply and demand?
Second, what is the appropriate production
volume and mix between product families?
The answers to these issues are heavily
dependent on the type of business
environment in which the business
operates. Knowledge in this area
encompasses:
1. The key linkages to the S&OP process
2. The different methods for balancing
supply and demand
3. The trade offs related to different
volume/mix combinations
4. How to carry out the S&OP process in
different business environments
B. Management Considerations: S&OP is the
process top management uses to control
and guide the business. It integrates the
multiple plans developed within a business
and provides direction for more detailed
short-term to medium planning. Knowledge
in this area encompasses:
1. How to evaluate alternative plans
2. How to assess the financial implications
of the plan
3. How to identify:
a. The various stakeholders in the
process
b. The planning horizon
c. The S&OP Process owner
4. How to establish:
a. Customer service levels
b. Inventory levels
c. Backlog levels
C. The Sales and Operations Planning
Process: There are several steps involved
in the process of establishing a sales and
operations plan. Effective S&OP requires
proper execution of each of these steps.
Knowledge in this area encompasses:
1. The various levels of management
involved and their roles
2. The five steps involved in S&OP
3. The various inputs and outputs of each
step
a. Month-end reports
b. Statistical forecasts
c. Field sales worksheets
d. Management forecasts
e. Capacity constraints
D. Developing and Validating the Production
Plan: A key output of S&OP is the
production plan. Effective S&OP requires
an understanding of this key planning

Master Planning of Resources

11

document. Knowledge in this area


encompasses:
1. How to develop and validate a
production plan in a variety of
production environments
2. How to carry out resource planning
3. How to assess the desirability of
alternative production plans
References: 1; 2; 4 (chapters 3, 12); 5
(chapters 1, 3, 4, 10, 13; Appendices A, D); 6
(chapters 1-6, 8-11, 14-17); 7 (chapters 1-3, 6,
Appendices A, G); 8 (chapter 13); 9 (chapters
1, 6)

III. Master Scheduling


This section addresses the processes of
translating higher level aggregate plans into
feasible schedules that operations and
suppliers can execute. Exam questions
covering this area will include elements related
to the following:
A. General Concepts and Purpose: The
master scheduling process disaggregates
production plan into an executable
schedule which links shipment of customer
orders to material management. Knowledge
in this area encompasses:
1. The role of master scheduling in the
planning and control hierarchy
2. The linkages with other planning
processes within the planning and
control hierarchy
3. The relationship between master
scheduling, capacity management, and
materials management
B. Linking the Master Scheduling Process to
the Business Environment: The mechanics
of master scheduling vary according to the
business environment. Nevertheless, in
every environment it is necessary to link the
master scheduling process with the
production plan and the S&OP process.
Knowledge in this area encompasses:
1. The relationship between the
manufacturing environment and
product structure
2. How to determine the level in the bill of
materials where the master schedule
should be developed.
C. Management Considerations: The master
scheduling process is the point where
higher level plans are converted into
executable actions. As such, it is essential
that the output of the process (i.e. the
master schedule) be consistent with the
overall goals of the organization.
Knowledge in this area encompasses:
12

CPIM Exam Content Manual

1. Senior executive responsibility related


to managing the master schedule
2. The role of the master scheduler
3. How to plan and coordinate changes in:
a. Inventory levels
b. Backlog
c. Capacity
d. Major customer orders
e. Time fences
f. Product or process designs
g. Suppliers
4. How to maintain the integrity of the
master schedule
5. How to deal with the consequences of
an unrealistic master schedule:
a. overloaded work centers
b. poor customer service
c. meaningless priority system
D. The Master Scheduling Process: The
process of developing a master schedule
involves numerous decisions related to the
mechanics and the selection of techniques
and tools. Effective master scheduling
requires significant insights into each of
these choices. Knowledge in this area
encompasses:
1. How to identify and quantify sources of
demand to be considered in the master
scheduling process
2. The mechanics of the master
scheduling process, including:
a. Disaggregating the production plan
b. Determining projected on-hand
balances and consuming the
forecast
c. Identifying available-to-promise
quantities using both the
cumulative and discrete methods
d. Working with time-phased order
records
e. Working with time-fence, and
rescheduling
f. Determining master scheduling lot
sizes
3. How to structure, interpret, and use a
bill of materials
4. Reviewing capacity requirements and
the role of rough-cut capacity planning
(RCCP)
5. Developing and using a final assembly
schedule (FAS)
E. Measuring Master Schedule Performance:
As with demand management and sales
and operations planning, measuring the
success of master scheduling is necessary
to using and improving the process.
Knowledge in this area encompasses:
1. How to validate the master schedule
2. Relating the master schedule to:

Delphi method
Demand filter
Demand time fence (DTF)
Design-to-order
Deviation
Distribution center
Distribution channel
Distribution network structure
Distribution of forecast errors
Double order point system

a. S&OP performance
b. Customer service levels
c. Marketing/sales performance
d. Purchasing performance
3. Maintaining data accuracy
References: 1; 2; 3 (chapters 1, 5, 6, 9, 11, 12);
4 (chapter 6); 5; 6 (chapters 3-5, 7, 9, 11, 13,
16, Appendix D); 7 (chapters 6, 7, Appendices
A, E, G); 8; 9 (chapters 6, 7)

Key Terminology
An understanding of the following list of terms is
recommended. The list is intended to be thorough
but not exhaustive. The candidate is also expected
to be familiar with the definitions of terms identified
in the curriculum outline and those definitions found
in the Key Terminology for Basics of Supply Chain
Management. A review of the list and study of the
topics in the outline may reveal other important
terms. Definitions of these terms can be found in
the APICS Dictionary, 12th edition, or in the
references listed in the bibliography.

Econometric model
Exponential smoothing forecast
Extrapolation

F
Feature
Finishing lead time
First-order smoothing
Focus forecasting
Forecast consumption
Forecast horizon
Forecast interval
Forecast management
Frequency distribution

H
A
Abnormal demand
Actual demand
Adaptive smoothing
Aggregate forecast
Aggregate plan
Allocation
Alpha factor

Hedge
Hybrid production method

I
Interplant demand

L
Lean production
Least-squares method
Level schedule
Life cycle analysis

B
Base series
Benchmarking
Bill of distribution (BOD)
Bill of resources
Bottom-up replanning
Bucket versus bucketless system
By-product

M
Manufacturing environment
Mass customization
Master planning of resources
Master schedule item
Master scheduler
Mean
Mean absolute percent error (MAPE)
Mean squared error (MSE)
Median
Mix forecast
Mixed-model scheduling
Mode
Modular bill of material
Moving average
Multilevel master schedule

C
Capable-to-promise (CTP)
Capacity planning using overall factors (CPOF)
Chase production method
Collaborative planning, forecasting, and
replenishment (CPFR)
Common parts bill of material
Consuming the forecast
Continuous production
Co-product
Correlation
Curve fitting
Customer relationship management (CRM)
Customer-supplier partnership

D
Decomposition
Delivery lead time

N
Noise
Normal distribution

O
Option
Option overplanning
Outlier
Overstated master production schedule
Master Planning of Resources

13

W
P:D ratio
Planning bill of material
Planning horizon
Planning time fence
Point of sale (POS)
Postponement
Priority
Probability
Probability distribution
Process flow production
Process manufacturing
Product configuration catalog
Product group forecast
Product line
Product load profile
Product mix
Product positioning
Product/service hierarchy
Production forecast
Production level
Production rate
Production schedule
Project manufacturing
Pyramid forecasting

Q
Qualitative forecasting techniques
Quantitative forecasting techniques

R
Random sample
Regression analysis
Resource profile
Running sum of forecast errors

S
Safety capacity
Sales promotion
Sample
Sampling distribution
Scatter chart
Seasonal index
Second-order smoothing
Service function
Single smoothing
Smoothing constant
Smoothing factor
Standard deviation
Strategic planning
Substitution
Super bill of material

T
Tactical planning
Time-phased order point (TPOP)
Time series analysis
Traceability
Trend forecasting models
Two-level master schedule

14

CPIM Exam Content Manual

Warehouse demand
Weighted moving average

Bibliography
All test candidates should familiarize themselves
with the following primary references. The
additional references provide further coverage of
some of the topic areas and can enhance a
candidates understanding of the body of
knowledge. The concepts and subjects covered in
the additional references, limited to the scope of the
curriculum outline, are part of the recognized
APICS body of knowledge and may be included in
examination questions. Also, candidates who have
not yet passed the suggested first moduleBasics
of Supply Chain Managementshould familiarize
themselves with the basic concepts of materials
management presented in Introduction to Materials
Management. All of these references are available
from the APICS Bookstore.

Primary References
1. APICS CPIM Master Planning of Resources
Reprints, October 2007.
2. APICS Dictionary, 12th ed., 2008.
3. Ross, D.F., Distribution Planning and Control,
2nd ed., Kluwer Academic Publishers, 2004.
4. Vollmann, T.E., W.L. Berry, D.C. Whybark, and
F.R. Jacobs, Manufacturing Planning and
Control Systems for Supply Control
Management, 5th ed., McGraw-Hill, 2005.
5. Wallace, T.F., and R.A. Stahl, Master
Scheduling in the 21st Century, T.F. Wallace &
Co., 2003.
6. Wallace, T.F., Sales & Operations Planning:
The How-to Handbook, 2nd ed., T.F. Wallace &
Co., 2004.
7. Wallace, T.F., and R.A. Stahl, Sales
Forecasting: A New Approach, T.F. Wallace &
Co., 2002.

Additional References
8. Proud, J.F., Master Scheduling, 3rd ed., John
Wiley & Sons, 1999.
9. Williams, B.R. Manufacturing for Survival,
Addison-Wesley, 1996.

Sample Questions
Following are 10 questions that are similar in format
and content to the questions on the Master
Planning of Resources exam. These questions are
intended for practicethat is, to enable you to
become familiar with the way the questions are
asked. The degree of success that you have in

answering these questions is not related to the


potential for success on the actual exam and should
not be interpreted as such.
Read each question, select an answer, and then
check your response with the explanation on page
42.
1. Which of the following demands is generally
NOT considered a part of demand
management?
(A) dependent demands
(B) interplant transfers
(C) service parts requirements
(D) branch warehouse requests

3. Producing exactly to demand should result in


(A) minimum change in inventory
(B) least total cost
(C) minimum capacity requirements
(D) maximum machine utilization
Lot Size: 30

Demand Time Fence: 3

On Hand: 15

Planning Time Fence: 7

Safety Stock: 6

6. The ability to notify a customer that his or her


order will be delayed because of a mechanical
breakdown is dependent on
(A) demand filters
(B) pegging
(C) safety stock
(D) outliers
7. Which of the following is LEAST likely to be
used as the value for alpha in an exponential
smoothing forecast when the company believes
recent history to be most important?
(A) .8
(B) .6
(C) .4
(D) .3

2. Normal uses of a master schedule include


which of the following?
I. To provide a means of planning future
production
II. To drive rough-cut capacity planning
III. To establish the production plan
(A) I only
(B) I and II only
(C) II and III only
(D) I, II, and III

Lead Time: 2

5. Using that same table, what is the projected


available balance for period 5?
(A) 30
(B) 35
(C) 5
(D) 7

PERIOD

Forecast

10

22

20

24

28

Customer Orders

26

15

30

Projected
Available Balance
Available-toPromise
Master Production
Schedule

8. The following are true of meaningful


performance measurements EXCEPT
(A) they relate to the organizations strategic
goals
(B) they relate principally to financial
performance
(C) they are communicated and understood at
all organizational levels
(D) they drive improvements in business
processes
9. The sales and operations planning process
brings together all of the following EXCEPT
(A) sales plan
(B) capacity requirements plan
(C) marketing plan
(D) new product development plan
10. The production plan is performed at which
level?
(A) product family
(B) work center
(C) end-item level
(D) product option level
(Answers listed on page 42.)

4. For the master schedule above, what is the


available-to-promise for period 4 if the discrete
method is used?
(A) 22
(B) 24
(C) 35
(D) 37

Master Planning of Resources

15

Detailed Scheduling
and Planning
Effective for January 1, 2008December 31,
2008, Exams.
EXAMINATION COMMITTEE
Ann K. Gatewood, CFPIM, CIRM (Chair),
Gatewood Associates
Sarah W. Klunk, CFPIM, CSCP, CIRM, C.P.M.,
Andrew Corporation
Donna M. Schmoll, CPIM, Flow International
Blair Williams, CFPIM, CSCP, Jonah, Polytechnic
University, Tech. Ed., Inc.

Scope of the Subject Matter


Please read the introductory materials in this
manual for essential information pertaining to the
examination.
The subject matter of Detailed Scheduling and
Planning includes inventory management, material
requirements planning, capacity requirements
planning, and procurement and supplier planning
for both producing goods and providing services.
Recognizing the importance of supply chain
management, this module also covers deployment
of supply chain strategies. The emphasis is on
applied- and higher-thinking questions.
Inventory planning concepts, policies,
methodologies, and techniques determine partstocking levels, order quantities, safety stocks,
forecasts, and handling and storage requirements.
Material requirements planning (MRP) deals with
the thousands of dependent demand parts and
interrelationships that require planning at any given
time; it also includes independent demand planning
for service, matching supply with demand,
managing demand at aggregate and disaggregate
levels, and other parts not encompassed by the
master production schedule (MPS). MRP combines
the following three principles:
1. calculation, as opposed to forecast, of
dependent demand for component items
2. netting of requirements for all items
3. time phasing (that is, adding the dimension
of timing to material requirements and
status data).
Capacity is defined as the ability to perform work to
produce goods or provide services. Capacity
requirements planning (CRP) provides a check that
the material plan is achievable based on existing
backlog and available capacity. Capacity
management encompasses planning, establishing,
measuring, monitoring, and adjusting levels of
16

CPIM Exam Content Manual

capacity so that sufficient capacity is available to


execute the master schedule and related materials
plan. CRP closes the loop based on the quantity
and scheduled output from MRP. For providing
services, uncertainty planning is also covered,
including variability and capacity.
Procurement and supplier planning encompass
planning and evaluation activities that companies
employ to qualify suppliers and establish effective
communication channels. Emphasis is on defining
mutual business needs, as well as defining the
materials, information, product, process, and quality
management and planning communications
required of the ongoing business relationship.
Detailed Scheduling and Planning translates
product-level plans and schedules generated at the
master planning level into requirements that can be
procured or produced, in all types of environments,
including process and service industries. This
process supports the strategies and objectives
established by the company, as constrained by lead
time, cost, equipment, personnel, or other
constraints. The subject matter therefore
encompasses anything required to bridge the
master planning area with the execution and control
area of the CPIM body of knowledge. This is
includes understanding the commonalities and
differences between producing goods and providing
services. Relevant strategy-level implications are
also considered.

Detailed Scheduling and Planning


Curriculum
This table identifies the three major Detailed
Scheduling and Planning subject areas. The
percentages provide a guideline for study purposes.
DIAGNOSTIC
PART
I

MAIN TOPIC
Planning Material
Requirements to Support the
Master Schedule

PERCENTAGE
OF EXAM
50%

II

Planning Operations to
Support the Priority Plan

30%

III

Planning Procurement and


External Sources of Supply

20%

Curriculum Outline
I. Planning Material Requirements to
Support the Master Schedule
This section addresses the techniques and
practices for managing item-level inventories

within manufacturing and the distribution


network.
A. Recognizing Techniques and Practices of
Inventory Management: This segment
covers item-level techniques used within
the detailed material planning process.
These include the effect of inventory
accounting decisions on material planning
calculations, as well as financial
management and accuracy. Cost objectives
that apply to inventory are evaluated with
respect to balancing the desired customer
service level with inventory reduction
techniques. Measures of inventory
performance are assessed.
1. Types and classifications of inventory
such as raw materials; work in process;
finished goods; maintenance, repair,
and operating supplies; excess;
surplus; inactive; obsolete; scrap; and
hard and perishable inventory in
service industriesare assessed from
their different requirements and impacts
on the planning processes. The scope
includes the entire product life cycle, as
well as different types of business
environments, including lean
manufacturing and services industries.
2. Order review methodologies such as
MRP, reorder point, visual review and
kanban/pull system triggers to MRP are
assessed with regard to different types
of inventory and inventory strategies.
This applies to both manual and
computerized processes.
3. Lot-sizing techniques, including the
effect of order quantity constraints, are
assessed as they apply to different
types of production and service
environments in support of inventory
investment strategies.
4. Safety stock techniques are assessed
as they apply to different types of
inventory in support of customer
service objectives.
5. Inventory valuation such as first in, first
out (FIFO), last in, first out (LIFO),
transfer, standard and actual cost, and
project and process cost, are assessed
as they affect inventory investment and
related replenishment and justification
decisions in various types of business
environments.
6. Inventory accuracy methodologies such
as cycle counting, physical inventory,
and use of dedicated containerization
are assessed as alternatives for
improving and sustaining part count
and inventory investment accuracy.

7. Inventory policies related to stocking


level, customer service and inventory
accuracy targets, trade-offs, and
emphasis in different types of business
environments are assessed.
8. Dependent and independent demand
are assessed as they relate to types of
inventory and sources of demand. This
includes demand planning for services
at aggregate and disaggregate levels.
9. Inventory performance such as
inventory turns, customer service, lean
techniques, inventory accuracy, and
others are assessed as they relate to
higher-level business metrics.
10. Inventory management in services
industries, including hard and
perishable inventories.
B. Identifying Information Used in the Material
Planning Process: This section describes
the inputs used in calculating requirements
for inventory items and their importance to
the detailed material planning process. Item
master data include information used for
planning time-phased requirements and
product costing. Bills of material define the
parent/component relationships and data
used by detailed material planning
processes to determine requirements for
replenishing inventory. Effective
management of engineering changes, open
order status, and master schedules are
critical to the effectiveness of the process.
1. Inventory data describe parts, define
current usage rates and stock
balances, and track historical demand
as required to support the policies,
methodologies, and techniques of the
material planning process.
2. Master schedule data describe types,
quantities, sources, priorities, and timephasing of product demand generated
as a result of the master planning
process as required to initiate part-level
planning.
3. Engineering data describe the product
structure and production routings
related to part interdependencies,
quantities, and lead times as required
for the aggregate planning, ordering,
safety stock, and engineering change
processes.
4. Data accuracy, timeliness, and
completeness are assessed as they
affect material planning decisions in
different types of environments.
C. Identifying the Desirable Characteristics of
the Detailed Material Planning Process:
This section addresses those

Detailed Scheduling and Planning

17

characteristics that enhance capabilities of


the users to plan and control system output.
Traceability of the sources of demand is
useful in evaluating the impact of system
action messages. Safety policies are used
to recognize the impact of uncertainty on
the planning process.
1. Design characteristics address the
required and desirable features and
functions that facilitate material
planning, establish requirements
traceability through pegging
relationships, and support other
decision functions and productivity
measures based on the type of
environment.
2. Performance characteristics address
the speed with which changes to the
material plan can be assessed and
alternative courses of action generated
and evaluated for feasibility.
3. Operational characteristics address the
ease of use, types of operator actions,
related system and process features,
and requirements for education and
training.
D. Mechanics of the Detailed Material
Planning Process: This section addresses
the use of planning data as the basis for
performing the gross-to-net calculations by
exploding bills of material to calculate the
time-phasing of inventory requirements.
The requirements are exploded level by
level, accounting for order quantity, safety
stock, and allocations at each level.
1. Initializing data accounts for inventory
already allocated to requirements,
establishing available stock balances
and scheduled receipts, and
establishing safety stock availability,
kanban quantity, cycle times, or the
need for replenishment.
2. The explosion process addresses the
mechanics of the gross-to-net
dependent demand aggregation
process. This can be via MRP, order
configurators, or other means, in
conjunction with various ordering and
safety stock techniques.
3. Generating time-phased requirements
creates a material plan that supports
the companys and suppliers needs for
longer-range planning and shorterrange tactical information. This is in
terms of firm-planned, planned, and
released orders at each bill of material
level and part. In a lean enterprise pull
systems are then integrated with
ERP/MRP systems.

18

CPIM Exam Content Manual

E. Maintaining the Validity of the Material


Plan: This section describes how the output
from the process recommends the
placement of supply orders. It also includes
making recommendations to replan open
supply orders to maintain the validity of
order priorities. The effects of planning
parameters upon the process are identified.
1. Maintaining priorities accounts for the
effect of changing demand priorities on
inventory allocation and replenishment
order timing.
2. Replanning accounts for
demand/supply mismatches through a
process of bottom-up replanning,
expediting, and de-expediting. This
includes related actions that
resynchronize the material plan with the
current material requirements.
3. Revising planning parameters
addresses the options of revising the
lead time, lot size, safety stock quantity,
kanban quantity, cycle times and
related parameters to reflect current
conditions and company strategy.
4. What-if analysis and modeling address
the tools, methodologies, and
techniques planners use to evaluate
viable alternatives within the constraints
of the material plan for producing
goods, and within the scheduling and
queuing constraints for providing
services
F. Interactions with Other Systems (i.e.,
closing the loop): This section identifies the
interrelationships between this process and
other planning and control subsystems. The
master schedule or actual customer orders
drive the detailed material planning
process. Output from the process provides
input to purchasing and production control
to release supply orders and control open
orders. Capacity planning is used to verify
capacity availability, which enables the
planner to balance projected loads with
planned available capacity for producing
goods and providing services
1. Business planning interactions close
the demand/supply loop with the
master planning, final assembly, and
configuration processes to ensure that
material availability matches demand
quantities, timing, and priorities. In a
services business, this includes
matching supply with demand.
2. Detailed scheduling and planning
internal interactions close the
material/capacity loop with the capacity
planning process to ensure that self-

manufactured materials and parts will


be available in the required quantities
and at the required time, and that
services will be provided as and when
they are required
3. Execution and control interactions close
the material/supply loop with the
purchasing processes to ensure that
purchased materials and parts will be
available in the required quantities and
at the required time. Material
planning/capacity processes are
integrated with customer demand, takt
time and pull systems to align shop
floor activity and WIP with planning and
scheduling systems.
References: 1; 2; 3 (chapters 4, 9-11); 5
(chapters 13, 14); 7 (chapters 2-5, 8, 9)

II. Planning Operations to Support the


Priority Plan
This section addresses the approaches for
balancing the material plans with available
internal resources and supporting activities. It
addresses the mechanics and application of
capacity requirements planning, constraint
management, and line and flow balancing.
A. Recognizing the Characteristics and
Techniques of the Detailed Capacity
Planning Process: This section addresses
the approaches for applying the process
within different production and services
environments. The system design
specifications and techniques applicable to
balancing capacity with demand and timing
availability with due dates are identified.
Techniques such as infinite and finite
capacity planning, constraint-based finite
schedulers and optimizers, load balancing
for scheduling manufacturing operations,
and establishing projected load on
manufacturing resources are described.
The segment gives attention to flow
production and process trains by
addressing process flow scheduling. Also
covered is uncertainty planning for
services, including variability and capacity.
B. Identifying Information Used in the Detailed
Capacity Planning Process: This section
addresses the use of work center and
routing data to schedule orders and
establish resource load by time period.
Efficiency and utilization are used to
determine the rated capacity of each work
center. The use of the material
requirements plan to establish release and
required dates for open and planned

manufacturing orders that provide the basis


of projecting resource load by time period
are described. The impact of unplanned
work and quality problems on resources is
identified. Also included are scheduling,
queuing, and throughput management in
service industries.
1. Capacity definition and availability, as
defined in terms of theoretical,
demonstrated, and rated, and as
qualified by efficiency and utilization
factors, are assessed in different types
of environments.
2. Sources of load, as they relate to
planned and released orders, repetitive
schedules, mill schedules, work in
process, level of rework, effect of scrap,
and effect of backlog, are assessed.
3. Definitions of process flows such as
routings, production models, and
process trains provide the sequence
and interdependencies of production
operations as they relate to capacity
and material timing.
4. Impacts of rework, scrap, and quality
on capacity management are assessed
with respect to how scheduling and
capacity can be affected by anomaly or
industry-specific conditions such as byproducts and recycled materials.
C. Identifying Techniques of the Detailed
Capacity Planning Process: This section
presents algorithms, simulation, and
modeling techniques that affect scheduling
and loading of manufacturing resources.
This section also covers how to apply
manufacturing concepts and techniques to
increase productivity in service industries.
1. Simulation and modeling techniques
enable a variety of scheduling and
loading conditions to be assessed with
respect to the type of production
processes experienced in various
environments.
2. Scheduling manufacturing operations
provides the feedback to material
planning required for synchronizing
order releases with available capacity.
3. Scheduling logistics operations
provides the feedback to purchasing
required for synchronizing order
receipts with material requirements.
4. Planning and scheduling horizons
provide the degree of forward planning
required to optimally or level-load
operations, and plan for capacity
reductions or increases as required to
support the volume of business and
customer service targets.

Detailed Scheduling and Planning

19

5. Queuing, sequencing, and scheduling


techniques provide insight into
scheduling principles, mechanics,
issues, and limitations of capacity
planning.
6. Infinite and finite loading techniques
have different impacts on order
releases with available capacity. Finite
loading techniques and algorithms
begin those used for advanced
planning and scheduling. Finite loading
algorithms help in understanding the
important sequencing rules.
D. Uses of the Detailed Capacity Planning
Process: This section identifies the basis
for adjusting the levels of capacity or load
to achieve scheduled required dates. The
use of safety capacity to compensate for
unplanned work is addressed. Tooling
requirements are scheduled to assure
availability at time of need. Managing
capacity is an important prerequisite for
successful operations planning and controls
systems.
1. Impacts of capacity management are
assessed based on the level of detail
required to produce effective load and
priority plans, including the use of
safety capacity in environments that
must accommodate unplanned load
variability.
2. Capacity planning outputs are
assessed in different environments as
they relate to formulating capacity
plans, determining tooling
requirements, and identifying overload
and underload conditions.
3. Methods of balancing capacity and load
are assessed in a variety of
environments as they relate to
rescheduling orders, splitting orders,
changing capacity through workforce
changes and subcontracting, and
modifying order quantities and
priorities.
E. Measuring the Performance of the Detailed
Capacity Planning Process: This section
identifies opportunities to assure success of
this process in achieving operational
objectives.
1. Planned workload to available capacity
measures the degree to which the
capacity planning process has
maintained the workload at or under the
available capacity.
2. Past-due load measures the degree to
which the capacity planning process
and conditions such as subcontracting

20

CPIM Exam Content Manual

and overtime have failed to support ontime performance.


3. Work in process measures the ability of
the production processes to minimize
work in process as a function of sales.
F. Interactions with Other Systems: This
section addresses how the detailed
planning processes interact with the other
operations planning, control, and related
processes.
1. Business planning interactions close
the demand/supply loop with the
master planning, final assembly, and
configuration processes to ensure that
capacity availability matches or
exceeds demand quantities, timing, and
priorities.
2. Detailed operations-planning internal
interactions close the material/ capacity
loop with the material planning process
to ensure that self-manufactured
materials and parts will be available in
the required quantities and at the
required time.
3. Execution and control interactions close
the capacity/supply loop with
production processes to ensure that
manufactured materials and parts will
be available in the required quantities
and at the required time. Pull systems
are integrated with MRP/ERP systems,
to align the shop floor activity and WIP
with customer demand and takt time.
References: 1; 2; 3 (chapters 5, 6, 14); 4; 7

III. Planning Procurement and External


Sources of Supply
This section addresses relationships with
suppliers and the approaches to communicating
requirements and schedules.
A. Establishing Relationships with Suppliers:
This section addresses the formation and
definition of the mutual relationship. The
focus is on the necessity of building trust
and establishing contractual relationships in
order to ensure a competitive advantage
and continuous improvement. Moving from
a supply chain strategy and vision to actual
techniques used by businesses today will
also be addressed.
1. A review of the principles of partnership
with suppliers addresses the varying
degrees of involvement and mutual
commitment of people in the company
and at the supplier. It considers
strategic alliances at a business level,
technical and commercial partnering at

a product level, and operational


partnering at a part or material level.
2. Relationship choices are made on the
basis of the companys needs for single
or multiple sources of supply. Related
ramifications in the initial supplier
selection, as well as such special
services as vendor-managed
inventories and on-site representation,
are addressed. Additional supply chain
links may exist, depending on the
supply chain strategy, including retail,
packaging and transportation
companies.
3. Effective communication between the
company and the supplier must take
into account the ongoing or unique
needs of the business relationship
specifically with regard to products,
processes, and technologies. Principles
and strategic elements of effective
communication are addressed, as are
such other considerations as language
differences, high engineering content,
commercial versus government
interests, and support by information
technologies. Data necessary for
collaboration includes risks, supply
chain inventories, and future demand.
CPFR techniques can be used to
formalize communication.
B. Techniques and Concepts for Supplier
Partnerships: This section explores the
formal and informal elements of
company/supplier interrelationships in
different competitive environments and
markets, and at different levels of
cooperation and involvement.
1. Product development and production
processes examine the value to
companies and suppliers of mastering
the process of simultaneous
engineering and the logistics of timephased and coordinated production and
delivery of goods and services.
2. Purchase order approaches consider
the specific means of establishing a
contract for the procurement and
delivery of materials, parts, products,
and services, as supported by the
planning and execution systems.
Alternative approaches such as
kanban, JIT/lean, blanket orders,
consignment, and special contracts are
evaluated with respect to the
companys production process and
operating philosophy. Techniques
deployed by large retail companies with
extensive supply chain needs are

included, including SRM principles and


activities, demand forecasting, direct
shipment, RFID and e-commerce.
Strategies and techniques of
outsourcing will also be covered.
3. Delivery approaches examine
alternatives used by companies to
shorten lead time and minimize their
delivery costs to the line. This includes
normal and expediting procedures,
batch size and packing, third-party
logistics, cross-docking, and point-ofuse and point-of-fit delivery.
4. Company billing processes address
alternatives and concepts that relate to
accounts receivable and accounts
payable. Representative contracts
consider free on board (FOB) point,
payment timing, holdback percentages,
methods of payment, penalties for late
payment, and so on.
5. The supplier rating system definition
encompasses the definition of supplier
evaluation and performance
requirements via quantitative methods,
as supported by the company through
on-time and clearly defined demand.
C. Implementing the New Relationship to
Support the Companys Operational
Objectives: This section addresses the
need to examine the quality of the supplier
partnership process. This includes any
related supplier or company training
requirements, with special emphasis on
modifications and continuous improvement.
1. Planning techniques encompass
procurement planning, new product
introduction, and engineering change
control as they relate to the effective
management of the companys
inventory investment, acceleration of
the supply chain and the total cost
relationship with suppliers.
2. Information used in the procurement
process is key to achieving the goals
defined for the company/supplier
relationship. The tools necessary for
effective information exchange are
surveyed. There is a special emphasis
on information technologies and
whether the two firms interact in the
most effective and flexible mode for
information exchange. Certain
obstacles must be overcome, including:
responsibility and accountability of all
partners, any migration to new
technology required, quality and
availability of data to be shared,
relationship management issues, and

Detailed Scheduling and Planning

21

decision-making procedures, such as


delivery timelines, quality targets, and
performance metrics.
3. Supplier and company-related training
addresses the need to provide training
and education in the areas of product
design, quality, production processes,
related technologies, and delivery and
accounting processes.
4. Supply chain partnerships bring many
benefits to all parties. Some of these
benefits include: Reducing inventory
while reducing stockouts. improved
customer service through customer
focus and responsiveness, B2B and Ecommerce capabilities, improved
visibility and value-chain forecasting,
cost reductions, damage and loss
prevention, elimination of redundant
supply chain assets and processes,
customer and marketplace insight,
access to or penetration of new
markets, reducing time to market,
streamlined management of order
fulfillment, and shorter overall lead
times

C
Calculated capacity
Capacity available
Capacity control
Central point scheduling
Collaborative Planning, Forecasting and
Replenishment (CPFR)

D
Decentralized inventory control
Decoupling inventory
Decoupling points
Dedicated capacity
Dedicated line
Drum-buffer-rope (DBR)

E
Effectivity date
Engineer-to-order
Excess capacity

F
First in, first out (FIFO)
Flexibility
Forward flow scheduling

G
Gateway work center

I
Idle capacity
Idle time
Inventory accounting
Inventory buffer
Inventory investment
Inventory policy

References: 1; 2; 3 (chapter 7); 5 (chapter 13);


6, 7

Key Terminology
An understanding of the following list of terms is
recommended. The list is intended to be thorough,
but not exhaustive. The candidate is also expected
to be familiar with the definitions of terms identified
in the curriculum outline and those definitions found
in the Key Terminology for Basics of Supply Chain
Management. A review of the list and study of the
topics in the outline may reveal other important
terms. Definitions of these terms can be found in
the APICS Dictionary, 12th edition, or in the
references listed in the bibliography.

J
Joint replenishment

L
Last in, first out (LIFO)
Load profile
Load projection
Lot sizing
Lot splitting
Low-level code

M
Machine center
Machine hours
Machine loading
Manufacturing order
Material-dominated scheduling
Min-max system
Mixed-flow scheduling

A
Action message
Activation
Advanced Planning and Scheduling
Allocation
Alternate routing
Availability
Available time

B
Budgeted capacity
Buffer management
Buffer stock
Business-to-business commerce (B2B)
By-product

22

CPIM Exam Content Manual

O
Operations sequencing
Order policy

P
Period order quantity
Perpetual inventory
Phantom bill of materials
Planned load
Probable scheduling

Process flow scheduling


Process manufacturing
Processor-dominated scheduling
Process train
Productive capacity
Protective capacity

S
Safety capacity
Safety lead time
Scheduled load
Scrap
Semifinished goods
Setup time
Shelf life
Shrinkage

Q
Quantity discount
Queue time

T
Rescheduling
Rework
Risk pooling

Target inventory level


Theoretical capacity

Y
Yield

MRP Grid
Many valid variations are used to display the MRP time-phased data. The following chart is the abbreviated grid
that is sometimes used in texts and work problems.

The MRP Grid


Technique

Order Quantity
Safety Stock
Allocated Quantity
Low Level Code
Lead Time

50
8
5
3

Gross Requirements
Scheduled Receipts
Projected Available
Net Requirements
Planned Order Receipts
Planned Order Releases

PERIODS

140

40

10

50

50

20

30

40

95

85

30
50
105

55

55
3
50

35

Bibliography
All test candidates should familiarize themselves
with the following primary references for this
examination. The additional references provide
further coverage of some of the topic areas and can
enhance candidates understanding of the body of
knowledge. Those concepts and subjects covered
by the additional references, limited to the scope of
the curriculum outline, are a part of the APICS body
of knowledge and are subject to examination
questions. Also, candidates who have not yet
passed the suggested first moduleBasics of
Supply Chain Managementshould familiarize
themselves with the basic concepts of materials
management presented in Introduction to Materials

50
Management. All of these references are available
from the APICS Bookstore.

Primary References
1. APICS CPIM Detailed Scheduling and Planning
Reprints, October 2001.
2. APICS Dictionary, 12th ed., 2008.
3. Arnold, J.R. Tony and S.N. Chapman,
Introduction to Materials Management, 6th ed.,
Prentice Hall, 2008.
4. Blackstone, J.H. Jr., Capacity Management,
South-Western Publishing Co., 2002.
5. George, Michael, Lean Six Sigma: Combining
Six Sigma Quality with Lean Production Speed,
McGraw-Hill, 2002.

Detailed Scheduling and Planning

23

6. Simchi-Levi, D., P. Kaminsky, and E. SimchiLevi, Designing and Managing the Supply
Chain, 3rd ed., McGraw-Hill, 2008.

(A)
(B)
(C)
(D)

line items shipped on time


total quantity filled
orders filled complete
ordering periods not out-of-stock

Additional References
7. Vollmann, T.E., W.L. Berry, D.C. Whybark, and
F.R. Jacobs, Manufacturing Planning and
Control Systems for Supply Chain
Management, 5th ed., McGraw-Hill, 2005.

Sample Questions
Following are eight questions that are similar in
format and content to the questions on the exam.
These questions are intended for practicethat is,
to enable you to become familiar with the way the
questions are asked. The degree of success that
you have in answering these questions is not
related to your potential for success on the actual
exam and should not be interpreted as such.
Read each question, select an answer, and check
your response with the explanation on page 42.
1. Service level =
Number of stockouts
1
Number of replenishment cycles
_____________________________________________________________________________________________________________________________

The expression above is used to measure


historical service levels in terms of a
percentage for which of the following statistical
control procedures?

24

CPIM Exam Content Manual

2. An organization using time-phased


requirements planning has the following net
requirements for an item:
WEEK
2
5
7
11
Lead Time
Unit Cost
Setup Cost
Carrying Cost

NET REQUIRED
350
1,200
1,000
1,500
1 Week
$1
$30
0.5% Per Week

If it is assumed that sufficient capacity is


available in weeks 1 through 11 to order any
amount in any period, which of the following is
the most economical ordering plan?
(A) Week 1
4
6
10
Order 350
1,200 1,000 1,500
(B) Week 1
6
Order 1,550
2,500
(C) Week 1
4
Order 350
3,700
(D) Week 1
4
10
Order 350
2,200
1,500

3. Lot Size:
On Hand:
Allocated:
Safety Stock:
Lead Time:

Lot-for-lot
500
0
0
4

The MRP Grid


Technique

Order Quantity
Safety Stock
Allocated Quantity
Low Level Code
Lead Time

PERIODS
1

Gross Requirements
Scheduled Receipts
Projected Available
Net Requirements
Planned Order Receipts
Planned Order Releases

500

100

300

200

400

100

400

The chart shows the gross requirements for an item in an MRP system. Stock on hand is 500, and there is
nothing on order. The item has a lead time of four periods and is being ordered lot-for-lot. Which of the
following would be the correct planned order release for the item?
(A) 100 in Period 1, 400 in Period 3
(B) 100 in Period 4, 400 in Period 6
(C) 100 in Period 5, 400 in Period 7
(D) 500 in Period 5

4. Calculation of rated capacity of a work center


includes consideration of which of the
following?
(A) efficiency and utilization
(B) utilization, setup, and run time
(C) utilization and available work time
(D) efficiency, utilization, and available work
time
5. All of the following are part of the CRP
process EXCEPT
(A) determining capacities of work centers
(B) determining the planned order release
(C) calculating load on each work center
(D) comparing work center load and capacity
6. To create an effective customer-supplier
partnership, we need to
(A) establish stability in schedules that enable
suppliers to react within their lead time
(B) evaluate the suppliers based on price and
then send the purchase order
(C) establish a delivery schedule with a
corresponding purchase order and send
them to the suppliers

(D) establish a preventive maintenance


program to avoid quality problems due to
machine troubles
7. A drill press operates at 50 percent efficiency
and 80 percent utilization. How many clock
hours must be worked to produce 80 standard
hours?
(A) 32
(B) 100
(C) 160
(D) 200
8. Which of the following might the planner use
to solve an overload problem?
(A) multiple setups
(B) alternate routings
(C) infinite loading
(D) adjust loading factors
9. Given the following purchase cost data for
product Z:
100 @ $10 = $1,000 (December 28)
10 @ $11 = $110 (January 3)
10 @ $8 = $80 (January 10),
Detailed Scheduling and Planning

25

if the company is using an annual standard


costing method and 110 units were sold on
January 11, the cost per unit for the sale is:
(A) $9.17
(B) $10.00
(C) $9.91
(D) $8.00

10. The major uses of the bills of material include


all of the following except:
(A) engineering change control
(B) planning of assemblies and components
(C) costing of the end product
(D) preventive maintenance schedule
(Answers listed on page 42.)

26

CPIM Exam Content Manual

Execution and
Control of
Operations
Effective for January 1, 2008December 31,
2008, Exams.
EXAMINATION COMMITTEE
Jorge E. Calaf, CPIM, CIRM (Chair), Kingfisher
Air Services, Inc.
John M. Burnham, PE, Ph.D., CFPIM, Professor
Emeritus Tennessee Technological University
Carol Davis, CPIM
Quentin K. Ford, CFPIM, Manufacturing Control
Associates, Inc.
Barry Griffin, Ph.D., CFPIM, Griffin Data
Perspectives, Inc.
Paul Schnsleben, Ph.D., Swiss Federal Institute
of Technology ETH Zurich

Scope of the Subject Matter


Please read the introductory materials in this
manual for essential information pertaining to the
examination.
Execution and Control of Operations (ECO)
encompasses the principles, practices, and the
techniques to schedule, control, measure, and
evaluate the effectiveness of operations. ECO
provides feedback about how well plans are being
executed and provides information for customers
and suppliers about the status of work in process
(WIP). The importance and emphasis of the
principles, approaches, and techniques addressed
are relative to the operations environment, the
labor environment, and the physical organization
of the facility, brought about through effective
people management and leadership.

Execution and Control of


Operations Curriculum
Three major subject areas define the discipline of
Execution and Control of Operations. The relative
importance of these topics will vary among
industries, but the percentages given in the
following table can be used as guidelines for study
purposes.

DIAGNOSTIC
PART

MAIN TOPIC

PERCENTAGE
OF EXAM

Prioritizing and Sequencing


Work to Be Performed

25%

II

Executing Plans, Implementing


Physical Controls, and
Reporting Results of Activities
Performed

40%

III

Performance Reporting and


Feedback

35%

Curriculum Outline
I. Prioritizing and Sequencing Work
to Be Performed
This section deals with interfaces for the
planning and execution of operation activities.
The focus will be on the techniques of
scheduling, monitoring, and controlling
operations in various environments and an
understanding of how facility layout
alternatives affect the operations scheduling
approach and the execution of processes.
A. Interfaces: This topic requires an
understanding of the interfaces between
execution systems and other key
functional elements in a closed-loop
system. These interfaces allow for input
from upstream planning activities and
outputs to the execution organization.
They also provide paths for feedback of
transactional data necessary to maintain
valid schedules throughout the system.
B. Operational Environment: Understanding
how the organizations environment
supports the processes, products and/or
services is an important aspect of
execution and control of operations.
Typical environments include lean,
repetitive, intermittent, projects,
remanufacturing, team-based groupings,
and service.
C. Understanding Schedule Development:
This segment covers how plans are
converted into operational tools that
provide the execution organization with a
timetable that includes suppliers and
distribution.
1. Understanding schedule development
requires knowledge about forward or
backward scheduling, mixed model or
rate-based scheduling, infinite versus
finite capacity, preventive
maintenance, and how material,

Execution and Control of Operations

27

labor, processes, or projects are


scheduled.
2. Management of the schedule is
essential throughout its execution.
Queues and priorities need to be
managed through effective use of
input/output controls. Techniques
such as lot splitting, overlapping, lot
size, runtime, and reducing
move/transit time may be used to
maintain priorities, including
equipment and labor elements.
3. Timely transactional feedback will
enable adjustments to the schedule
when applicable.
References: 3 (chapters 1, 4-7, 9-10);
5 (chapters 2, 4-6, 8-9, 11); 6 (chapters 9-11,
13, 15-16, 18); 7 (chapters 1, 5-7); 8 (chapters
1, 13-14, 16, 18-19)

II. Executing the Plans, Implementing


Controls, and Reporting Results of
Activities Performed
This section encompasses the application and
control of internal and external resources to
accomplish near-term plans. It also addresses
implementing quality initiatives, continuous
improvement plans, and physical control of
inventories.
An understanding of the different methods for
authorizing and reporting work accomplished
and resources consumed is outlined in this
section. Knowledge of the approaches and
techniques used to reduce or eliminate nonvalue-added activities within the ECO
processes is required.
A. Understanding the Operations of Push
Systems: This section focuses on the
methods for authorizing work and
reporting work accomplished in a system
where work is authorized by means of
work orders and/or schedules.
1. Authorizing activities include
processes that assess the material,
labor and capacity availability, as well
as the sequence, routing, and release
of orders to work centers.
2. Execution activities include staging of
materials and work in process
throughout the supply chain.
3. Reporting activities include
documentation and communication of
the operations results, work in
progress, work completed, parts
scrapped, and items that have been,
or need to be, reworked.
28

CPIM Exam Content Manual

B. Understanding the Operations of Pull


Systems: This section focuses on the
methods for authorizing work and
reporting work accomplished, when
applied in a system where work is
authorized by means of readily
understood visual signals. An
understanding of lean methodology is
necessary.
1. Visual techniques are used to
authorize the movement and/or
production of materials or the
provision of service.
2. Understanding what execution
activities are required in a pull system
as well as the effect on work in
process is necessary to determine the
number and character of visual
signals.
3. Understanding the reporting activities
in a pull system is essential, and may
include visual elements or information
relating to corrective action, scrap,
rework, order completion, and/or
shipments.
C. Communicating Internal Information
consists of transaction reporting or
collecting data at a count point for
scheduling, finance, engineering, quality,
and other internal users.
D. Communicating External CustomerSupplier Information focuses on the
methods of sharing information
throughout the supply chain from the
suppliers supplier to the customers
customer.
E. Material Flow and Location: This section
deals with the methods of managing
operations, inventory and material flow to
achieve the goals of the organization.
Recognition and understanding of the
environment is required.
1. Material location and flow concerns
include containerization, point of use
or manufacture, and material
handling.
2. Material flow and movement is
dependent upon the environment and
can be different in push or pull
environments.
F. Quality Initiatives, Eliminating Waste, and
Implementing Continuous Improvement
Plans requires an understanding of the
philosophies, approaches, concepts, and
practices of quality management and lean
concepts.
1. Quality initiatives include process
improvement tools that help to obtain
Six-Sigma process reliability,

capability, and standardization.


Included in the set of process
improvement tools are statistical tools
that diagnose chronic and sporadic
problems alike, as well as managerial
tools useful for small group
improvement activities.
2. The practice of continuous improvement
and waste elimination includes improved
housekeeping and workplace organization,
paperwork and transactions reductions.
Timely problem resolution at the point of
occurrence is fundamental to this process.
Lead-time, lot size, move time, and
throughput time reductions are additional
benefits.
References: 3 (chapters 5, 7, 10); 4 (chapters
1, 3, 5-6, 8-11, 16); 5 (chapters 1, 3-12, 14); 6
(chapters 5, 9, 11, 13, 15-17, 19); 7 (chapters
4-7); 8 (chapters 9, 14, 17-18)

III. Performance Reporting and


Feedback
This section requires an understanding of the
approaches and techniques used to evaluate
performance and collect data for effective
feedback. The structure of data collection and
concepts of analysis are defined, along with
commonly used techniques to evaluate an
operations performance. Data captured
enables comparison of performance to preset
standards, past performance, objectives,
goals, and actual results. Many levels of the
organization use data to identify problems and
initiate corrective action
An understanding of specific evaluation
criteria for operations, data collection
techniques, analyses, and reporting methods
is necessary. The ability to apply this
knowledge results in timely recognition of the
need for corrective action, in addition to the
use of performance information for costing,
budgeting, and calculating the impact of
proposed changes throughout the
organization.
A. Quality Management Processes monitor
and evaluate the effectiveness of quality
management initiatives as well as
continuous improvement efforts. Typical
decisions resulting from the above can
include changes in standards or
tolerances, modifications to, or acquisition
of machinery, improved methods or
measurement approaches, material or
supplier changes, and revised resource
strategies.

1. Understanding the effects of process


variation, whether the variation is
special or common will determine the
breadth, character, cost, and
effectiveness of ensuing operational
decisions.
2. Statistical process control seeks to
diagnose and correct undesirable
variation in processes through
identification, updates, maintenance,
or relevant data.
3. Process capability determines
whether the variation in a process is
sufficiently controlled in order to
ensure that the process meets, and
will continue to meet, quality and
output objectives. Collection,
maintenance, and updates of the
relevant data is key.
B. Measuring Supplier Performance: An
understanding of the reporting
requirements to support both the
organization and supplier goals is
necessary.
1. Specifications and standards are
used to test the acceptable range of
values of product and process
variation.
2. Quality audits are an evaluation
process that an organization or
external qualifying body uses to
decide whether supply chain partners
can maintain process quality at levels
sufficient to ensure final product
quality, including delivery and
aftermarket performance in addition to
the basic unit.
3. Inspection of supplier product on
receipt may not be required if supplier
qualification has been achieved. If the
supplier has not been qualified,
sampling and full inspection are two
of the techniques that may be
employed.
4. Performance metrics to support
operations are monitored and
reported.
C. Measuring Operations Performance: This
section covers the collection and
maintenance of data to support
operational decisions.
1. Data sources and requirements are
affected by the environment, and
require an understanding of how to
measure the results.
2. Performance reporting is vital to
measure operations.
D. Cost Management Process: Knowledge of
the different costing methods and their

Execution and Control of Operations

29

impacts on the organization is essential.


An understanding of actual and standard
costs is necessary, as well as the
knowledge of causes of variances.
Individuals and work groups need to
understand how they are measured and
how operational variables affect the costvolume-profit relationship.
1. Cost management and control of
operations can be accomplished by a
variety of methods.
2.
Periodic audits and cost controls
ensure that operations remain in compliance and
variances are identified.
References: 3 (chapters 1, 9, 12); 4 (chapters
2-3, 5, 8-11, 13-19, 22-23); 5 (chapters 1, 3-4,
8-14); 6 (chapters 4-5, 16-17); 7 (chapters 1,
4); 8 (chapters 5-6, 9, 17-18, 21); 9 (chapters
2-3, 7, 10-11)

Critical characteristics
Critical path method
Critical point backflush
Critical ratio
Critical-to-quality characteristics (CTQ)

D
Decision matrix
Delivery schedule
Design for manufacture and assembly (DFMA)
Design-measure-analyze-improve-control
(DMAIC)
Design of experiments (DOE)
Direct costs
Downtime
Drum-buffer-rope (DBR)

E
External setup time

F
Failsafe techniques
Failure mode effects analysis (FMEA)
Feeder workstations
Five whys (5 Ws)
Floor stocks
Flowchart
Flow control
Flow rate
Focused factory

Key Terminology
An understanding of the following list of terms is
recommended. The list is intended to be thorough,
but not exhaustive. The candidate is also
expected to be familiar with the definitions of
terms identified in the curriculum outline and those
definitions found in the Key Terminology for
Basics of Supply Chain Management. A review of
the list and study of the topics in the outline may
reveal other important terms. Definitions of these
terms can be found in the APICS Dictionary, 12th
edition, or in the references listed in the
bibliography.

G
Gantt chart
Gateway work center
Green belt
Group technology (GT)

H
Histogram
House of quality (HOQ)
Hypothesis testing

A
Acceptable quality level (AQL)
Acceptance sampling
Actual cost system
Allocation
Alternate operation
Analysis of variation (ANOVA)
Andon
Anticipated delay report
Assignable cause
Attribute data
Average outgoing quality limit (AOQL)

B
Backflush costing
Back scheduling
Black belt
Block scheduling

C
Capability study
Cause-and-effect diagram
Certified supplier
Common causes
Count point

30

CPIM Exam Content Manual

I
Inbound stockpoint
Indirect costs
Internal setup time
Interoperation time

K
Kit

L
Lean enterprise
Lean metric
Lean production
Level schedule
Linearity
Line balancing
Lot splitting
Lot traceability

M
Machine-limited capacity
Manufacturing execution systems (MES)
Master black belt
Mixed-model production
Move card

T
Network planning
Non-value-added

Throughput
Time series
Time standard
Total quality control (TQC)

O
One less at a time
Operation due date
Operation duration
Operation/process yield
Operations scheduling
Operations sequencing
Operation start date
Order release
Outbound stockpoint
Overlapped schedule
Overload

P
Pareto analysis
Plan-do-check-action (PDCA)
Point-of-use delivery
Point-of-use storage
Post-deduct inventory transaction processing
Pre-deduct inventory transaction processing
Preventive maintenance
Primary work center
Process capability
Process costing
Production card
Program evaluation and review technique
(PERT)
Project costing

Q
Quality costs
Queue time

R
Random causes
Rate-based scheduling
Released order
Rework

S
Scheduled downtime
Scheduling rules
Scrap factor
Setup time
Shop packet
Shortest processing time rule (SPT)
Single-minute exchange of die (SMED)
Supplier, Input, Process, Output, Customer
(SIPOC)
Slack time rule
Special causes
Specification
Staging
Standard containers
Standard cost accounting system
Statistical quality control (SQC)
Stockless production
Supplier scheduling
Synchronized production

V
Variable costing
Visual control

W
Workplace organization

Y
Yield

Bibliography
All test candidates should familiarize themselves
with the following primary references for this
examination. The additional references provide
further coverage of some of the topic areas and
can enhance candidates understanding of the
body of knowledge. Those concepts and subjects
covered by the additional references, limited to
the scope of the curriculum outline, are a part of
the APICS body of knowledge and are subject to
examination questions. Also, candidates who
have not yet passed the suggested first module
Basics of Supply Chain Managementshould
familiarize themselves with the basic concepts of
materials management presented in Introduction
to Materials Management. All of these references
are available from the APICS Bookstore.

Primary References
1. APICS CPIM Execution and Control of
Operations Reprints, 2000.
th
2. APICS Dictionary, 12 ed., 2008
3. Blackstone, J.H. Jr., Capacity Management,
South-Western Publishing Co., 2002.
4. Gryna, F.M., Jurans Quality Planning &
Analysis for Enterprise Quality, 5th ed.,
McGraw-Hill, 2007.
5. Sandras, W.A., Just-in-Time: Making It
Happen, John Wiley & Sons, 1989.
6. Vollmann, T.E., W.L. Berry, D.C. Whybark, and
F.R. Jacobs, Manufacturing Planning and
Control Systems for Supply Chain
Management, 5th ed., McGraw-Hill, 2005.

Additional References
7. Feld, W.M., Lean Manufacturing: Tools,
Techniques, and How to Use Them, St. Lucie
Press, 2001.
8. Fogarty, D.W.N., J.H. Blackstone Jr., and T.R.
Hoffmann, Production and Inventory
Management, 2nd ed., South-Western
Publishing Co., 1991.
Execution and Control of Operations

31

9. Garrison, R.H., E.W. Noreen, and P.C.


Brewer, Managerial Accounting, 11th ed.,
McGraw-Hill, 2004.

Sample Questions
Following are 10 questions that are similar in
format and content to the questions on the exam.
These questions are intended for practicethat
is, to enable you to become familiar with the way
the questions are asked. The degree of success
that you have in answering these questions is not
related to your potential for success on the actual
exam and should not be interpreted as such.
Read each question, select an answer, and then
check your response with the explanation on page
44.
1. Which of the following is the most viable
approach to automating repetitive assembly
processes?
(A) simplify the processes prior to automation
(B) concurrently establish visual inspection
stations and automation implementation
(C) implement automation throughout the
processes simultaneously rather than
work cell by work cell
(D) apply automation to the final assembly
processes first
2. Cause-and-effect analyses are made with the
help of which of the following?
(A) fishbone chart
(B) statistical process control method
(C) critical path method
(D) Pareto analysis
3. Data collection using bar-code technology can
provide which of the following?
I. Ability to determine throughput time in
real time
II. Simplified data recording
III. Increased data accuracy
IV. Cost reductions
(A) I and III only
(B) I, II, and IV only
(C) II, III, and IV only
(D) I, II, III, and IV
4. Company X, a job shop, is a supplier to
Company Y, which produces a product
continuously. The companies agree to begin a
Lean program to reduce lead times through all
operations in both facilities. To implement the
program, which of the following actions should
be taken?
I. Company X should reduce the shipment
quantities to Company Y.
32

CPIM Exam Content Manual

II. Company X should reduce its


manufacturing lot size.
III. Company Y should transfer the inspection
procedure to Company X.
(A) I only
(B) II only
(C) I and II only
(D) I, II, and III
5. Part number 762-4521 is being processed
as an order for a lot of 50 pieces due on
day 200. The lot is currently in queue at
Operation 3. The facility is currently
running one eight-hour shift. Based on
routing and status information for part
number 762-4521 below, what is the
priority for Operation 3 on day 195
calculated on the basis of slack time (in
days) per remaining operation?

OPERATION

STANDARD
HOURS PER
PIECE

1
2
3
4
5

0.05
0.15
0.10
0.20
0.02

(A)
(B)
(C)
(D)

PIECES
PRODUCED
THROUGH
DAY 195
51
50
0
0
0

-3.70
0.58
1.00
3.00

6. Which of the following actions is most


appropriate for resolving a capacity shortage
at the initial or gateway work center that was
detected during order release?
(A) add a shift at the gateway work center
(B) expedite in-process orders
(C) hire additional staff for the work center
(D) reschedule orders due for release
7. Which of the following is a significant factor in
determining the level of work-in-process
inventory when a pull system is employed?
(A) number of open shop orders
(B) quantity of parts represented by each
signal
(C) number of workstations in the process
(D) takt time required for the process
8. A benefit of non-significant part numbers is:
(A) one part number can be used for several
similar items
(B) they can include numbers and letters
(C) responsibility can be assigned

(D) they support faster and more accurate


entry by people
9. Capture of inventory transactions is
(A) more accurate when done promptly
(B) not required when using a bar-code
system
(C) best done by data entry clerks
(D) more important for perishable stock

10. Which of the following would be used to take


a group of information and organize it in a way
that new trends or patterns of information are
easily recognized?
(A) run chart
(B) cause-and-effect diagram
(C) Pareto chart
(D) flow chart
(Answers listed on page 44)

Execution and Control of Operations

33

Strategic
Management of
Resources
Effective for January 1, 2008December 31,
2008, Exams.
EXAMINATION COMMITTEE
Stephen Chapman, Ph.D., CFPIM (Chair), North
Carolina State University
Craig Gustin, Ph.D., CFPIM, CIRM, CSCP, CGR
Management Consultants
Frank Montabon, Ph.D., CPIM, CIRM, Iowa State
University
Rebecca Morgan, CPIM, Fulcrum
ConsultingWorks, Inc.
Andy Nourse, CPIM, BD Biosciences, Discovery
Labware
David Rivers, CFPIM, CIRM, CSCP, Axtin

Scope of the Subject Matter


Please read the introductory material in this
manual for essential information pertaining to the
examination.
The subject matter in the Strategic Management
of Resources module includes higher-level
thinking on strategic planning and implementation.
This requires an understanding of how market
requirements drive the resources and processes
of all organizations.
The strategic management of resources has the
potential to dramatically increase any
organizations competitive position. Operations
strategy is the foundation upon which operations
planning and control decisions should be based.
Strategic decision-making significantly affects the
organization and often requires widespread
changes. To contribute to this effort, it is important
to understand how an organization develops its
strategic goals, and how it configures its systems
and technologies to address strategic objectives.
This examination focuses on the relationship of
existing and emerging systems and concepts to
the strategy and the functions related to
operations planning and control. Understanding
how Strategic Management of Resources relates
to the CPIM body of knowledge as defined by the
other CPIM modules is important.

34

CPIM Exam Content Manual

While the curriculum outline provides an overview


of the key concepts and subject matter, it is also
important to have the ability to move from a
modular view to an integrated systemic view of
the CPIM body of knowledge. Effective strategic
decision making requires an ability to understand
the dynamic interrelationship of concepts, the
nonlinear nature of complex systems, and an
ability to see the long-term implication of
decisions. By itself, this outline cannot sufficiently
capture these important aspects of the strategic
decision-making process.

Strategic Management of
Resources Curriculum
This table identifies the three major Strategic
Management of Resources subject categories.
The percentages provide a guideline for study
purposes.

DIAGNOSTIC
PART

MAIN TOPIC

PERCENTAGE
OF EXAM

Aligning the Resources with


the Strategic Plan

46%

II

Configuring and Integrating the


Operating Processes to
Support the Strategic Plan

23%

III

Monitoring Performance and


Implementing Change

31%

Curriculum Outline
I. Aligning the Resources with the
Strategic Plan
This area focuses on developing knowledge
of those factors that can change an
organizations competitive position and
understand distinctive competencies. This
knowledge, matched with a clear
understanding of the strategic environment,
structure, and infrastructure, will enable
organizations to more effectively align
resources with the strategic plan to create a
competitive advantage.
A. Strategic Issues: In this section, a broad
range of strategic issues is explored to set
the stage for identifying the environment
within which the operations professional
must perform. An understanding of the
factors that can change an organizations
competitive position is required.
1. The need for a company strategy
Understand the integration of
processes with overall company
objectives.

2. The strategic planning model


Identify and select strategic
components.
3. The need for distinctive
competenciesUnderstand how an
organization can select and display
distinctive competencies to support
strategic objectives.
B. Choices Affecting Operations Structure:
This section focuses on the operational
aspects of the strategic plan. Choices
include an understanding of product
decisions and product characteristics
related to volume and variety. The
structural choices governing operations
strategy are also covered. Structural
choices typically involve capital
investments that are difficult to reverse.
1. Operations strategy introduction
Understand the concepts and
purpose of an operations strategy.
2. Product and process decisions
Understand the effect of production
volume and variety on the
configuration of a factory.
3. Product life cycleRecognize the
interactions of product life cycle and
operations strategy.
4. Structural decisionsAlign operations
strategy and the structural choices of
organizational design, capacity
strategies, facility decisions, and
technology.
C. Choices Affecting Operations
Infrastructure: This section focuses on the
operational aspects of the strategic plan.
Choices include an understanding of the
types of infrastructure decisions of
operations systems configuration,
workforce, and organizational
development. Infrastructural choices are
typically easier to reverse.
1. Infrastructure choices and
productivityUnderstand the impact
of infrastructure choices on
organizational productivity.
2. Workforce involvementUnderstand
the alignment of human resource
policies and practices with strategies.
3. Organizational designUnderstand
the alignment of organizational design
with strategies.
4. Quality systemsUnderstand how
the quality tools and processes are
used to implement the strategic
choices.
5. Information systemsUnderstand
how information systems influence
and support operations strategy.

References: 1; 2; 3; 4; 5 (chapters 3-4);


6 (chapters 1, 2, 4, 6, 8-12, and supplement);
7 (chapters 1-7, 10); 8 (chapters 1-7);
9 (chapters 1-2); 10 (chapters 10-12)

II. Configuring and Integrating the


Operating Processes to Support
the Strategic Plan
The elements that comprise the production
and inventory management (P&IM) system
must work in an integrated manner if the
system is to operate effectively. It is important
to understand the interrelationships between
these elements and be able to analyze and
configure P&IM systems for specific strategic
and operational situations.
A. Implementing the Strategic Plan: This
section focuses on aligning operating
processes with selected strategies to
support the strategic business objectives.
It addresses the fit of approaches and
methods to specific strategic and
operational situations and the alignment
and integration with those strategies.
1. Operations management system
Understand the relationships between
the operational management
processes and the other business
choices.
2. Operations management system
configurationUnderstand the
characteristics of the operations
management processes that are
consistent with each position on the
volume and variety matrix.
3. Supporting dimensions of competitive
advantageUnderstand how the
characteristics of the operations
planning and control process support
each dimension of competitive
advantage.
4. Supporting combinations for
competitive advantageUnderstand
how processes can be combined to
support specific combinations of
volume and variety positions and
dimensions of strategic advantage.
B. Emerging Concepts and the Supply
Chain: This section focuses on the
linkages between the strategic plan, the
structure of the organizations supply
chain, and how those structures are
evolving. It addresses the fit of methods
to specific strategic and operational
situations as well as the alignment and
integration with the other supply chain

Strategic Management of Resources

35

management processes and other


emerging concepts.
1. Supply chain management and
scopeDefine the supply chain
(design, planning, execution, control,
and monitoring), and the value chain.
2. Global sourcing and outsourcing
Understand global sourcing,
outsourcing, insourcing, and other
supply chain management issues.
3. Technology and the supply chain
Recognize how the choices and
development of information
technology affect the supply chain.
4. Time-based competitionUnderstand
how time-based competition and
cycle-time reduction improve
customer response time and provide
competitive advantage.
5. Strategic implications and
relationships of concepts such as
lean, Just-in-Time (JIT), agile, theory
of constraints (TOC), total quality
management (TQM), and six sigma.
6. Value stream mappingRecognize
how value stream mapping provides
prioritization for improvements in the
value stream.
C. Configuring and Integrating Design and
Development and Cost Management
Processes: This section focuses on two
sets of processes that are typically
outside the scope of operations but
influence the effectiveness of operations.
Understanding the scope of the
processes and the demands placed on
them is important. Design and
development employ a set of processes
that extends from the initial investigation
of a concept through the selection of
concepts to implement and support the
resulting products and processes through
their life spans. Cost management
processes support external reporting of
financial results, asset valuation,
operational control, and product costing.
These processes capture, organize, and
retain data used in making decisions that
influence current operations and the
future competitive position of the
organization.
1. Design and development processes
Recognize tools that can be
employed to improve these
processes, understand how the
processes can provide competitive
advantage, and understand the
management of product and process
changes.

36

CPIM Exam Content Manual

2. Cost management processes


Understand cost composition and its
relationships with technology and
process choices, cost accounting
methods and their application, asset
valuation methods and their
application, and methods for financial
settlement with customers and
suppliers.
References: 1; 2; 3; 6 (chapters 1, 6, 7, 10,
13, and supplement)

III. Monitoring Performance and


Implementing Change
Recognizing, understanding, and applying the
methods used to implement change of various
operational management systems is
important. These systems of change are
required to enable people to either
continuously improve or revolutionize the
processes in which they are involved. By
properly monitoring and measuring operations
and taking appropriate action, cost, quality,
cycle time, and related goals and objectives
can be achieved.
A. Project Management: This section
focuses on the concepts, approaches,
and methods for defining, selecting, and
managing projects. It addresses the
process of defining the current situation
and the proposal for the target situation,
justifying the proposal, and planning and
controlling the project. It also addresses
the approaches and techniques for
evaluating proposed projects and
selecting them for implementation.
1. The project evaluation and selection
processUnderstand the various
methods of determining a projects
value and justifying the adoption of a
project.
2. The project planning process
Understand the various items that
make up a project plan, including a
statement of work, work breakdown
structure, project schedule,
responsibility matrix, resource
requirements, and budget.
3. The project implementation phase
Understand that this phase includes
the selection of a project
management structure, the project
team formation, the scheduling of the
work, team dynamics, and managing
the schedule and budget.
4. The closeout phaseUnderstand that
this phase includes documentation of

the project, a post-implementation


audit, customer approval, closing the
operations, and the final report.
B. Measuring Organizational Performance:
This section focuses on the various
methods of developing, deploying, and
evaluating measurement processes that
are central to effective management and
leadership of the organization. It
addresses concepts that are fundamental
to the effective use of measurement
strategies and techniques of
organizational performance, as well as
the selection of measurements that are
appropriate in the alignment of specific
strategic and operational situations to the
expected results.
1. Fundamental concepts of
measurementIdentify why we
measure what we measure.
2. Organizational measurements and
the design of measurement
systemsUnderstand appropriate
measurements of the various
functional areas such as
manufacturing, finance, engineering,
sales, marketing, and overall
organizational performance.
3. Strategic implications of
measurementUnderstand that what
we are measuring aligns with what we
should measure and the performance
of the organization matches the
interpretations of the various
performance measurements.
C. Change Management: This section
addresses the elements of successful
organizational change and sound
methodologies to gain management and
employee commitment. The scope of
change management begins at the point
where the areas for change have been
identified and extends through the
institutionalization of the change.
1. Organizational change concepts
Recognize why organizations change,
the phases of organizational change,
and individual and group responses to
change.
2. Change processUnderstand the
steps in successful organizational
change and tools that can assist the
organization in executing each step.
3. How to lead changeRecognize the
differences between management
and leadership and the roles of
leaders in implementing
organizational change.

4. Preparing employees for change


Recognize the tools that can help
prepare employees for change and
the ways to gain commitment to
change efforts.
References: 1; 2; 3; 4; 5 (chapters 1-2, 5-9,
11, 13); 6 (chapter 14 and supplement);
7 (chapters 8, 14); 9; 10 (chapters 1-9, 14)

Key Terminology
An understanding of the following list of terms is
recommended. The list is intended to be thorough,
but not exhaustive. The candidate is also
expected to be familiar with the definitions of
terms identified in the curriculum outline. A review
of the list and study of the topics in the outline
may reveal other important terms. Definitions of
these terms can be found in the APICS Dictionary,
12th edition, or in the references listed in the
bibliography.

A
Absorption costing
Acceptance sampling
Activity-based cost (ABC) accounting
Activity-based management (ABM)

B
Backward integration
Baseline measures
Benchmarking
Benchmark measures
Break-even chart
Break-even point
Bullwhip effect
Business process reengineering (BPR)

C
Capacity strategy
Cash conversion cycle
Cash-to-cash cycle time
Competitive advantage
Computer-aided design (CAD)
Computer-aided engineering (CAE)
Computer-aided manufacturing (CAM)
Computer-assisted software engineering
(CASE)
Computer-integrated manufacturing (CIM)
Concurrent engineering
Continuous manufacturing
Core competencies
Core process
Corporate culture
Critical chain
Critical path method (CPM)

D
Database management system
Decision support system (DSS)
Strategic Management of Resources

37

Delivery lead time


Depreciation
Design for manufacturability
Discounted cash flow

Project costing
Prototyping

Q
Quality costs
Quality function deployment (QFD)

E
Economic value added
Enterprise resources planning (ERP)
Expert system

Flexibility
Focused factory
Forward integration

Residual income
Return on investment (ROI)

F
Strategic drivers
Sunk cost
Surge capacity
Synchronized production
Systems thinking

G
Gantt chart
Generally accepted accounting principles
(GAAP)
Global measures
Group technology (GT)

H
House of quality (HOQ)
Hurdle rate

Theory of constraints accounting

V
Value chain
Value stream
Variable costing
Vertical integration
Voice of the customer (VOC)

Information system architecture


Internal rate of return
ISO 9000 series standards
ISO 14000 series standards

Supplemental Glossary

Job enlargement
Job enrichment

The following concept, not found in the APICS


Dictionary, will be helpful in preparing for the
Strategic Management of Resources examination.

J
K
Knowledge-based system

L
Lean enterprise
Learning curve
Learning organization
Life-cycle costing
Local measures

N
Net present value
Network planning

O
Operations strategy

P
Participative design/engineering
Participative management
Payback
Performance measure
Pilot test
Preventive maintenance
Process capability
Process control
Process costing
Process focus
Product focus
Product profiling
Program evaluation and review technique
(PERT)
38

CPIM Exam Content Manual

Insourcing: A strategic decision to utilize internal


(in-house) rather than external processes or
resources.

Bibliography
All test candidates should familiarize themselves
with the following primary references for this
examination. The additional references provide
further coverage of some of the topic areas and
can enhance candidates understanding of the
body of knowledge. Those concepts and subjects
covered by the additional references, limited to
the scope of the curriculum outline, are a part of
the APICS body of knowledge and are subject to
examination questions. Also, candidates who
have not yet passed the suggested first module
Basics of Supply Chain Managementshould
familiarize themselves with the basic concepts of
materials management presented in Introduction
to Materials Management. All of these references
are available from the APICS Bookstore.

Primary References
1. APICS CPIM Strategic Management of
Resources Reprints, July 2005.

2. APICS Dictionary, 12th ed., 2008.


3. Garrison, R.H., E.W. Noreen, and P.C.
Brewer, Managerial Accounting, 12th ed.,
Irwin/McGraw-Hill, 2007.
4. Harvard Business Review: Strategic
Management of Resources Readings, 2000.
5. Meredith, J.R., and S.J. Mantel Jr., Project
Management: A Managerial Approach, 5th
ed., John Wiley & Sons, Inc., 2003.
6. Slack, N., and M. Lewis, Operations Strategy
with APICS Special Edition Supplement,
Prentice Hall, 2003.

Additional References
7. Evans, J.R., and W.M. Lindsay, The
Management and Control of Quality, 6th ed.,
South-Western Publishing Co., 2004.
8. Hayes, R.H., G. Pisano, D. Upton, and S.C.
Wheelwright, Pursuing the Competitive Edge,
John Wiley & Sons, 2005.
9. Project Management Institute Standards
Committee: A Guide to the Project
Management Body of Knowledge, PMI, 2000.
10. Senge, P.M., The Dance of Change, Currency
Doubleday, 1999.

Sample Questions
The following ten questions are similar in format
and content to the questions on the exam. These
questions are intended for practicethat is, to
enable you to become familiar with the way the
questions are asked.
The degree of success that you have in answering
these questions is not related to your potential for
success on the actual exam and should not be
interpreted as such. Read each question, select
an answer, and check your response with the
explanation on page 45.
Questions 1 through 4 are based on the following
information:
Barnyard Plows is a multinational
manufacturer of farm implements. Using its
own dealer network, it sells anything you can
drag behind a tractor in the mature,
competitive farm implement aftermarket. The
firm has decided to centralize its operations in
Belgium. Each of the current plants in France,
Germany, Switzerland, Italy, and Sweden will
be converted from manufacturing many end
products and components to manufacturing
high volumes of only a subset of the
components. Final assembly of all end
products will be done in Belgium, but without
dedicated assembly lines. In addition, the

product line will be standardized, eliminating


older, unprofitable products and reducing the
number of variations in the remaining
products. Marketing believes that the standard
product line will cover 95 percent of the
markets needs (anything you can drag behind
a tractor).
1. The changes in manufacturing are being
made to address which of the following orderwinning criteria?
(A) quality conformance
(B) price/cost
(C) delivery reliability
(D) flexibility
2. Which of the following types of manufacturing
processes is being implemented at the
component manufacturing locations?
(A) jobbing
(B) batch
(C) line
(D) continuous
3. The firm currently has an excellent closedloop MRP II system and is implementing Justin-Time (JIT) in all the component plants.
Which of the following activities is LEAST
appropriate for the component plants?
(A) workplace organization
(B) quality programs
(C) pull systems
(D) work order control
4. At the assembly plant in Belgium, all of the
following techniques are appropriate EXCEPT
(A) sales and operations planning
(B) mixed-model scheduling
(C) forward integration
(D) resource requirements planning
5. In which of the following areas can
manufacturing contribute the most to the
product development process?
(A) product demand forecasting
(B) manufacturing process innovation
(C) bill of material structuring
(D) product technology selection
6. The selection of a flexible manufacturing
system (FMS) should be considered only
when
(A) a high level of reactive control has been
established by plant personnel
(B) a high level of responsiveness to product
mix and feature changes is required
(C) the number of parts assigned for
production can be reduced to a small
number

Strategic Management of Resources

39

(D) the savings in manufacturing costs


exceed the cost of the new system
7. In an engineer-to-order company, which of the
following is essential to the successful
implementation of rough-cut capacity
planning?
I. Education of system users and
management in capacity management
concepts
II. Successful implementation of a material
requirements planning system
III. Accurate work-in-process inventory and
job routing data
IV. A method of associating planned
production with the amount of capacity
available
(A)
(B)
(C)
(D)

I and III only


I and IV only
II and IV only
I, II, and III only

8. The key advantages to be gained from group


technology are
(A) reduced lead times and lower work-inprocess inventory
(B) increased flexibility in scheduling and
more reliable customer service
(C) reduced machine run times and more
reliable customer deliveries
(D) faster introductions of new products and
improved product quality

40

CPIM Exam Content Manual

9. When considering the adoption of new


manufacturing technology, a firm should take
the following actions. What is the most
effective sequence of the following actions?
I. Prepare a requirements definition.
II. Study the technology in a similar
environment.
III. Get input from workers in the current
process.
(A)
(B)
(C)
(D)

I, II, III
II, III, I
III, I, II
III, II, I

10. Which of the following measures properly


considers the financial impact of
subcontracting a family of parts that are
currently being fabricated?
(A) benchmarking
(B) direct labor efficiency
(C) baselining
(D) total factor productivity
(Answers listed on page 45.)

Answers to Sample
Questions
Note: References to the curriculum outline
appear in parentheses.

Basics of Supply Chain


Management
1. D (IE) An effective supplier/customer
partnership is based on a mutual commitment
to produce a profitable product that meets
market demand. This includes sharing
problem-solving activities to ensure that the
product is delivered. (A) is incorrect because
the customers requests may not be profitable
or possible for the supplier to meet. (B) is
incorrect because the supplier may be able to
make the part less expensively and/or of
higher quality than can the customer. (C) is
incorrect because EDI does not ensure
feasible schedules for either the customer or
the supplier.
2. A (IIB) An order winner is defined as a
product characteristic that causes customers
to choose the product over that of a
competitor. (B) is incorrect because it does
not suggest that the product characteristic
leads to success in the market. (C) and (D)
are incorrect because they refer to profitability
and sales dollar volume rather than to the
specific characteristics of the product that lead
to an individual sale.
3. A (IID) The primary purpose of forecasting
is projecting product demand. Three of the
secondary uses of forecasting are (B) in
estimating safety stock, (C) in determining
resource requirements, and (D) in loading the
master production schedule.
4. D (IIIC) The production plan is used to
determine rates of production for product
families. (A) is incorrect because customer
service level depends on the availability of a
specific product for a customer on a specific
requested delivery date. (B) is incorrect
because gross sales volumes are estimated
during business planning, not during
production planning. (C) is incorrect because
the production plan is established from the
business plan, not vice versa.
5. B (IIID) Available-to-promise quantities are
used during order promising to commit master
schedule product to individual customer
orders. (A) is incorrect because the projected
available balance is used by the master
scheduler to balance total demand (including

6.

7.

8.

9.

10.

forecast) with the master schedule supply. (C)


is incorrect because safety stock is inventory
maintained to protect against fluctuations in
demand or supply and is not necessarily a
part of order promising. (D) is incorrect
because forecast demand may be used to
plan the master schedule; however, order
promises are based on the existing master
schedule.
A (IIID) Through employee empowerment,
statistical process control of an established
stable process should be monitored by the
individuals directly performing this process.
(B), (C), and (D) are inappropriate answers
because these individuals oversee production
but do not directly perform the operations.
C (IVA) Safety stock is used to protect
against variations in supply and/or demand.
(A) is incorrect because cycle stock is
consumed by demand and replenished
through replenishment orders. It is the
inventory carried to cover expected demand.
(B) is incorrect because common stock is a
financial term for ownership in a company. (D)
is incorrect because seasonal stock is
inventory built up to smooth production in
anticipation of a peak seasonal demand.
D (IVA) Dependent demand items are raw
materials, component parts, and
subassemblies. The demand for these items
is calculated (through the bill of material)
based on the demand for their parent items.
(A) is incorrect because finished goods
represent independent demand. (B) is
incorrect because service part demand is not
necessarily dependent on a parent demand.
(C) is incorrect because price sensitivity of a
product is not a factor in the dependent
demand concept.
D (IIID) In backward scheduling, the due
date is initially used to calculate the schedule.
The early start date (A) and the late start date
(B) represent actual results achieved during
production. (C) is derived by scheduling
backward from the due date, using
replenishment lead time.
B (IIIC) The bill of material is used during
MRP to guide the explosion process, thereby
calculating gross requirements for
components. (A) is incorrect because the net
requirements calculation depends on existing
replenishment orders for the components. (C)
is incorrect because guiding the assembly
process is not part of the MRP calculation. (D)
is incorrect because the bill of material does
not specify lot-sizing information. This is
generally carried on the item master.

Answers to Sample Questions

41

Master Planning of Resources


1. A (IA) Demand management is the function
of recognizing and managing all of the
demands for products to ensure that the
master scheduler is aware of them. Generally,
dependent demands are managed through
material requirements planning. The other
demands listed are independent demands,
and these fall within the function of demand
management.
2. B (IIIB) The master schedule is a plan of
future production and drives the rough-cut
capacity plan. The production plan is used as
a basis for the master schedule, rather than
the master schedule being used as a basis for
the production plan.
3. A (IID) Producing exactly to demand will
maximize capacity requirements. Capacity will
be required to meet the highest demand
levels, yet this capacity (including machine
time) will go unused when the demand is low.
Costs will be high due to low utilization of
capacity. However, producing exactly to
demand will minimize average inventory,
because the inventory level will not rise during
slack periods.
4. B (IIID) ATP for period 4 is the MPS of 30
for that period, less the customer orders of 6
for that period. Because another MPS of 30
will occur in Period 5, demand for that period
is not considered by the ATP for period 4.
5. B (IIID) PAB for period 5 uses the greater of
customer orders or forecast since it is after
the demand time fence. The prior period PAB
of 35, plus the MPS of 30, less the orders of
30, results in a PAB of 35 for period 5.
6. B (IIIC4) Pegging ties source of supply and
demand, thus allowing a manufacturing
reschedule to be tacked to the customer order
affected by change.
7. D (IB) A higher value for alpha is used when
one wants the forecast to be more responsive
to recent actual demand values.
8. B (IC) While many traditional measurements
were financially based, proactive metrics
should consider a much wider scope than
simply financial performance.
9. B (IIA) The capacity requirements plan is
created after both S&OP and master
scheduling are complete.
10. A (IIC) The production plan, an output of the
S&OP process, specifies the overall
manufacturing output level to be produced,
usually as a monthly rate for each product
family.

42

CPIM Exam Content Manual

Detailed Scheduling and


Planning
1. D (IE) The percentages of line items
shipped on time (A), of the total quantity filled
(B), and order filled complete (C) are all wrong
because the formula does not include any of
these statistics. The formula (D) includes the
number of stockouts that have occurred
during some number of replenishment cycles
(i.e., lead times). One minus the proportion of
replenishment cycles in which a stockout
occurs is the percentage of ordering periods
not out-of-stock, i.e., service level.
2. D (IA)
Total Cost = Setup Cost + Carrying Cost
Setup Cost = Number of Setups Cost per
Setup
Carrying Cost = Units Unit Cost
Number of Periods Carrying Cost per
Period
The costs are calculated as follows:
A. Setup Cost = $120 = $30 4
Carrying Cost = $0
Total Cost = $120
B. Setup Cost = $60 = $30 2
Carrying Cost = $48
= (1,200 $1 x 3 .005)
+ (1,500 $1 x 4 .005)
Total Cost = $108
C. Setup Cost = $60 = $30 2
Carrying Cost = $55
= (1,000 $1 2 .005)
+ (1,500 $1 6 .005)
Total Cost = $115
D. Setup Cost = $90 = $30 3
Carrying Cost = $10
= (1,000 $1 2 .005)
Total Cost = $100

3. A

(ID)

Lot Size:
On Hand:
Allocated:
Safety Stock:
Lead Time:

Lot-for-Lot
500
0
0
4

The MRP Grid


Technique

Order Quantity
Safety Stock
Allocated Quantity
Low Level Code
Lead Time

PERIODS
1

Gross Requirements
Scheduled Receipts
Projected Available
Net Requirements
Planned Order Receipts
Planned Order Releases

500

100

100

300

200

400

400

100
100
100

400
400

400

------------------------------------------------------------------------------------------------------------------------------------------------

4. D (IIB) Setup and run time are used to


calculate required capacity. The rated
capacity = time available efficiency
utilization.
5. B (ID) The scheduled receipts and planned
orders receipts are inputs to CRP. The
planned order release is used in the MRP
explosion.
6. A (IIIA) Partnerships require a commitment
to stability within the agreed-upon response
parameters. (B) and (C) are short-term and
tactical in nature. (D) is really something that
needs to be done without any regard to
partnering.
7. D (IIB) Standard hours = (clock hours)
(utilization efficiency). Therefore, clock
hours = (standard hours)/(utilization
efficiency) = 80/(0.80 0.50) = 200 hours. (A)
is incorrect because 32 = (standard hours)
utilization efficiency. (B) and (C) are
incomplete applications of the correct formula,
dividing standard hours by utilization alone to
arrive at 100 (B), and by efficiency alone to
arrive at 160 (C).
8. B (IIC) If the planner had an alternate route,
it could be used to distribute or divide the load
between two choices, which makes (B) the
correct answer. Multiple setups (A) add to the
load, making it an incorrect choice. Infinite
loading (C) detects the overload but does not
solve the overload problem. Loading factors

should reflect reality. Adjusting accurate


loading factors (D) to address an overload will
only hide reality from the planner; therefore, it
is also incorrect.
9. B (IA) Standard costing is a method of
setting a new standard based on the last cost
for the unit at the end of the year. The only
exceptions would be if the suppliers had set a
new cost for the upcoming year. (A) is wrong
because of average cost. (C) is wrong
because this is LIFO. (D) is wrong because
this is the last cost paid for the item.
10. D (IB) Reference Arnold, pages 85-86.
(A) is a major use of the BOM because
engineering sometimes changes the design of
the products and a record is required to know
what to change.
(B) is a major use of the BOM because when
planning the end item the required
components and their relationship must be
used to determine the plans start and due
dates.
(C) is a major use of the BOM because to cost
the end item all components and their
quantities and costs must be identified.
(D) does not use the BOM.

Answers to Sample Questions

43

Execution and Control of


Operations
1. A (IB) The effectiveness of automation
depends on improving the manufacturing
process before you begin automating the
process. The approach is to first simplify the
process, then to automate the process in
order to eliminate work and improve quality.
(B), (C), and (D) are incorrect. (B) improving
inspection does not improve the process,
overlooking the first step of simplifying the
process. (C) (D) implementing automation
prior to simplifying the process significantly
increases the chance of making mistakes.
2. A (IIIA) The fishbone chart is a diagram of
the possible causes of a problem. The causes
are determined with the aid of brainstorming
techniques. The diagram resembles a fish
skeleton.
(B), (C), and (D) are incorrect. SPC (B)
focuses on the continuous monitoring of a
process. The critical path method (C) is a
technique used to plan and control the
activities of a project. Pareto analysis (D) is a
technique to rank order the relative frequency
of occurrences.
3. D (IIIC) Increased data accuracy is the
correct answer because bar coding eliminates
the possibility of typing or transposition errors
when entering data. (A), (B), and (C) are
individually incorrect because they do not
include all of the options.
4. D (IIF) I, II, and III are all ways to reduce
lead time and improve performance.
I. Reduced shipment quantities, requiring
greater frequency of delivery, will reduce
queues in Company Y and also reduce intransit inventory between X and Y.
II. Reduction of lot sizes in Company X is a
way to reduce queues in Company X and
therefore reduce total lead time.
III. Elimination of receiving inspection reduces
lead time and, by inspecting at the source (or
closer to it), downstream problems can be
reduced. (A), (B), and (C) are all incorrect,
because each one excludes one or two
correct options.
5. C (IC) The slack time per remaining
operation formula is:
Date Due Date Now Standard
Processing Days Remaining
The result is divided by the number of
remaining operations.

44

CPIM Exam Content Manual

OPERATION

STD.
HOURS

PIECES
PRODUCED

1
2
3
4
5

0.05
0.15
0.10
0.20
0.02

51
50
0
0
0

PROCESS
REMAINING
HOURS
0
0
5
10
1

The priority of Operation 3 is calculated as


follows:
(200 - 195) - (16 hrs. 8 hrs. per day)
3

5-2
3

=1

6. D (IIA) Because the capacity shortage was


detected during order release and at a
gateway work center, the corrective action
needs to be implemented in the very nearterm. Since the capacity shortage was not
detected at the higher levels of capacity
planning, it is likely to be a short-lived
imbalance. Rescheduling orders that have not
yet been released can be done quickly and
will provide immediate relief at the gateway
work center. (A), (B), and (C) are incorrect.
(A) and (C) Adding a shift or hiring employees
would increase capacity, but it is unlikely that
either could be accomplished in time to
address the short-term and immediate
problem. (B) Expediting in-process orders will
have no effect on the capacity available or
capacity required at a gateway work center.
7. B (IIB) The level of work-in-process
inventory when a pull signal is employed is a
function of the number of pull signals and the
quantity represented by each pull signal. (A),
(C), and (D) are incorrect. (A) Shop orders are
characteristic of a push system and are not
relevant in a pull system. (B) The number of
workstations in the process determines the
minimum work-in-process level, but does not
determine the total work-in-process level. (D)
The takt time determines the rate at which the
process needs to operate. It does not
determine the work-in-process level.
8. D (IIE) Nonsignificant part numbers are
usually smaller and therefore will require less
time and be more accurate for data entry. (A),
(B), and (C) are incorrect. (A) Under no
circumstance should a single part number be
used for several items, similar or not. (B)
Significant and nonsignificant part numbers
may include numbers and letters (C) You can
assign responsibility with either significant or
non-significant part numbers.
9. A (IIE) The sooner the transactions are
captured, the more accurate the information
will be. (B), (C), and (D) are incorrect. (B) A
bar-code system facilitates data entry, but
does not eliminate data entry. (C)
Transactions are best done by the personnel

who are most familiar with the type of


transactions one is working with, not data
entry clerks. (D) All inventory transactions are
important, not just for perishable stock.
10. A (IIIA) Run chart shows the variation trends
in a group of information over time. (B), (C),
and (D) are incorrect. (B) A cause-and-effect
diagram is used to collect brainstorming ideas
to establish categories of potential causes to
an effect. (C) A Pareto chart is used to sort
data into groups from most significant to least
significant. (D) Activities that are included on a
flow chart identify the relationship to one
another and do not identify trends or patterns.

Strategic Management of
Resources
1. B (IA) The changes described in the
scenario (centralizing assembly in one plant,
focusing each component plant, and
standardizing the product line) point to
price/cost as the dominant order winner. The
standardization and plant reorganization
decrease the flexibility and dont contribute
significantly to quality conformance. Neither of
these changes improves delivery reliability,
and may make transportation (hence delivery)
more problematic. However, these changes
do allow focus on component manufacture
(with larger volumes), which should achieve
economies of scale exceeding the increased
transportation costs.
2. C (IB) The component plants are each
focused on a narrower range of parts.
Because the demand for all components
throughout Europe is consolidated, the
volumes are higher, positioning them as a line
manufacturing process type. Note that the
assembly plant in Belgium would be
positioned as a batch process type.
3. D (IC) The benefits of orderly work places
and high quality are independent of the use of
MRP II and JIT. The demand for all
component plants is created by MRP IIcalculated requirements coming from the
assembly plant in Belgium. Within the
component plants, pull control is appropriate,
and the release of specific work orders is not.
4. C (IC) The assembly plant certainly requires
sales and operations planning (with the
attendant resource requirements planning).
The diverse models will be mixed-model
scheduled to accommodate the diverse needs
of the market and to balance the demands
placed on the component plants. The
widespread and diverse nature of the market
precludes forward integration.

5. B (IIC) The manufacturing organization is


not involved in forecasting demand. Its role in
developing the bills of material is subordinate
to the engineering organization. Although the
manufacturing organization is involved with
the selection of appropriate product
technologies, such choices are governed
much more by manufacturing engineering and
design engineering. By contrast,
manufacturing has a very significant role in
suggesting and developing innovative
processes.
6. B (IB) An FMS is typically quite expensive
to purchase, to install, and to maintain. It must
be justified by its high flexibility and quick
changeover. A high level of reactive control
doesnt support such a system (and may work
against it). If the number of parts could be
reduced to a small number, then alternative,
cheaper processes are available (such as
work cells), eliminating answer (C). Although
benefits greater than cost (answer D) are
always desirable as far as this line of thinking
goes, it ignores the significant ongoing
running, maintaining, and programming costs
of an FMS. Even if savings do not exceed
costs, the strategic need for flexibility will
offset this shortfall.
7. B (IC) Rough-cut capacity planning (RCCP)
is often successful in plants that do not have
MRP-based scheduling or fully accurate
routings through the shop. RCCP uses its own
bills of resources; hence IV is needed, as is
appropriate training (I).
8. A (IB) Group technology-based work cells
tend to collapse operations formerly assigned
to several work centers into one tightly
coupled cell. This does not affect actual run
time, but considerably reduces queue, move,
and wait times, hence reducing lead time and
work in process. Scheduling may be less
flexible because of the need to associate
parts with particular cells. There are no direct
or significant effects on new product
introductions or quality.
9. C (IIIA) A new process shouldnt be
considered before understanding the present
one; hence, III is the first step. Preparing the
requirements definition narrows the universe
of possible technologies. When one or a few
have been identified, appropriate users of
such technologies may be visited.
10. D (IIIB) Many costs may be incurred in
changing parts from internal manufacture to
subcontract. Labor factor productivity neglects
engineering, transportation, energy, and other
potential factors. Baseline and benchmark
procedures might be used to establish goals
for either fabrication or subcontracting, but do

Answers to Sample Questions

45

not contribute to the financial analysis. Total


factor productivity considers all the factors in
combination with each other.

46

CPIM Exam Content Manual

Das könnte Ihnen auch gefallen